New classes are starting soon - secure your spot today!

Mathcamp 2023 Qualifying Quiz Math Jam

Go back to the Math Jam Archive

This Math Jam will discuss solutions to the 2023 Mathcamp Qualifying Quiz. Mathcamp is an intensive 5-week-long summer program for mathematically talented high school student. More than just a summer camp, Mathcamp is a vibrant community, made up of a wide variety of people who share a common love of learning and passion for mathematics.

Copyright © 2024 AoPS Incorporated. This page is copyrighted material. You can view and print this page for your own use, but you cannot share the contents of this file with others.

Facilitator: AoPS Staff

devenware 2023-05-23 19:00:22
Hello and welcome to the Canada/USA Mathcamp Qualifying Quiz Math Jam!
winstonDeGreef 2023-05-23 19:00:37
hi
achyutksingh 2023-05-23 19:00:37
hihi
MathWinner121 2023-05-23 19:00:37
Glad to be here
BabaLama 2023-05-23 19:00:37
hi
devenware 2023-05-23 19:00:39
Before I introduce our guests, let me briefly explain how our online classroom works.
devenware 2023-05-23 19:00:44
This room is moderated, which means that all your questions and comments come to the moderators. We may share your comments with the whole room if we so choose.
devenware 2023-05-23 19:01:01
Canada/USA Mathcamp is an intensive five-week-long summer program for high-school students interested in mathematics, designed to expose students to the beauty of advanced mathematical ideas and to new ways of thinking. You can learn more about Canada/USA Mathcamp here: www.mathcamp.org
devenware 2023-05-23 19:01:06
Many AoPS instructors, assistants, and students are alumni of this outstanding program!
devenware 2023-05-23 19:01:14
Each year, Mathcamp creates a Qualifying Quiz, which is the main component of the application process. If you haven't already seen it, you can find the 2023 Quiz problems at https://www.mathcamp.org/qualifying_quiz/ . In this Math Jam, the following Mathcamp staff (all members of this year's Quiz committee) will discuss the problems and their solutions:
devenware 2023-05-23 19:01:54
Tim Black (timblack) has taught Mathcamp classes on topics ranging from complexity theory to board games, plus a lot of ways to avoid doing calculus. He's been at camp as either a camper or staff most summers since 2005. He also spent many years teaching math and computer science at the University of Chicago.
devenware 2023-05-23 19:02:06
Dan Gulotta (dgulotta) has been a camper, mentor, and visiting speaker at Mathcamp, most recently in 2021. He is currently a postdoc at Boston University, studying number theory.
Yunny 2023-05-23 19:02:24
:)
ily 2023-05-23 19:02:24
It's starting!!! :O :grin:
MathWinner121 2023-05-23 19:02:35
Hello!!!!!
EpicBird08 2023-05-23 19:02:35
yay!
devenware 2023-05-23 19:02:40
Let's turn the room over to Tim and Dan now to get us started!
timblack 2023-05-23 19:02:58
Hello everyone!
dgulotta 2023-05-23 19:03:08
Hi everyone!
timblack 2023-05-23 19:03:08
A big thanks as always to @devenware and the AoPS team for hosting us, and to all of you for being here!
MathWinner121 2023-05-23 19:03:19
Hello, Tim and Dan!
WangJY01 2023-05-23 19:03:19
Hi!
akmathisfun2020 2023-05-23 19:03:19
hello!
bobjoebilly 2023-05-23 19:03:19
hi
Curious_Droid 2023-05-23 19:03:19
hi
alex4010 2023-05-23 19:03:19
Hi
timblack 2023-05-23 19:03:28
Tonight, we'll be working through solutions to the Mathcamp 2023 Qualifying Quiz. This Math Jam is likely to be interesting mostly to students who applied to Mathcamp and worked through the Qualifying Quiz on their own, but if you didn't, you're of course still welcome to stick around and see some interesting problems!
timblack 2023-05-23 19:03:38
To follow along, you may want to have the Qualifying Quiz open in another window: https://www.mathcamp.org/qualifying_quiz/current_quiz/
timblack 2023-05-23 19:03:46
Today, we'll just be talking about the Quiz. If you have questions about Mathcamp itself, or the application process, you'll find lots of info on our website (e.g., at https://www.mathcamp.org/about_mathcamp/).
timblack 2023-05-23 19:04:05
The Quiz problems are written by Mathcamp alumni, staff, and friends each year, and the solutions we'll be walking through today are a collaboration by lots of Mathcamp staff (with good ideas from the applicants, too!).
timblack 2023-05-23 19:04:20
If you applied this year, I strongly recommend having your own solutions open. That way, you can reply more quickly to the questions we ask of the room. And we're expecting you all to pitch in to the solutions!
timblack 2023-05-23 19:04:34
We'll be posting the transcript from tonight on https://www.mathcamp.org/qualifying_quiz/solutions/ within the next few days, so no need to worry if you have to miss part of the event.
timblack 2023-05-23 19:04:56
Students can use LaTeX in this classroom, just like on the message boards. Specifically, place your math LaTeX code inside dollar signs. For example, type:



We claim that \$\frac{1}{2} + \frac{1}{3} = \frac{5}{6}\$.



This should give you:



We claim that $\frac{1}{2} + \frac{1}{3} = \frac{5}{6}$.
timblack 2023-05-23 19:05:08
As @devenware pointed out: this chat room is moderated. That means your messages go only to us, and we will choose which to pass on, so please don't be shy to contribute and/or ask questions about the problems at any time (and we'll do our best to answer).

We've got a lot to cover, so let's get started! I'll dive right into Problem #1 to kick us off.
timchen 2023-05-23 19:05:25
$\frac{1}{2}+\frac{1}{3}\neq\frac{5}{6}.$
timblack 2023-05-23 19:05:28
:O
timblack 2023-05-23 19:05:43
1. You have a funny calculator with only two buttons: $+1$ and $\times 2$. The first button adds $1$ to the current number, the second multiplies it by $2$. For each nonnegative integer $n$, what is the shortest sequence of buttons that will get you from $0$ to $n$? (As with all problems on the Qualifying Quiz, make sure to justify your answer with a proof.)
timblack 2023-05-23 19:05:57
For this first problem especially, instead of just typing out the solution, I want to go through the problem solving process with you.
timblack 2023-05-23 19:06:06
To save some typing, I'm going to write + to represent the $+1$ button and * for the $\times 2$ button. I'll write Z to mean "start with zero."

What number do we end up with if we press Z+*++*+ ?
TheBeast5520 2023-05-23 19:06:39
9
vrondoS 2023-05-23 19:06:39
9
Technodoggo 2023-05-23 19:06:39
$9$
angie. 2023-05-23 19:06:39
9
impromptuA 2023-05-23 19:06:39
9
HappyBunny 2023-05-23 19:06:39
9
timblack 2023-05-23 19:06:42
That's right:
$0 + 1 = 1$,
$1 \times 2 = 2$,
$2 + 1 = 3$,
$3 + 1 = 4$,
$4 \times 2 = 8$,
$8 + 1 = 9$.
So Z+*+++*+ $\rightarrow 9$.
timblack 2023-05-23 19:06:59
There it took seven button presses to get to 9 (the Z doesn't count). Is this the shortest sequence to get from zero to 9?
ThousandthStar 2023-05-23 19:07:21
No
winstonDeGreef 2023-05-23 19:07:21
no
spm269 2023-05-23 19:07:21
no
wilsonstmartin 2023-05-23 19:07:21
no
timblack 2023-05-23 19:07:29
What's a shorter sequence?
rithek.shankar 2023-05-23 19:07:54
There is Z+***+
JigglypuffDaBest 2023-05-23 19:07:54
Z+***+
pierrecai 2023-05-23 19:07:54
Z+***+=9
BrianSun 2023-05-23 19:07:54
Z+***+
timblack 2023-05-23 19:08:02
Nice. Z+***+ $\rightarrow 9$, and that's only five presses.
timblack 2023-05-23 19:08:10
In this case, to get from $0$ to $9$, we started by pressing the + to get $1$, then we pressed * as many times as we could without going over 9 (in this case, three times), then we pressed + until we got to $9$ (in this case, just once).
timblack 2023-05-23 19:08:56
This seems like a promising method to me: after we get to $1$, it feels good to press * as many times as we can, because that lets us quickly jump to bigger and bigger numbers. For example, Z+++++++ is only $7$, while Z+****** is already $64$. But is this really the best method? Again, the method is: press + once, then press * as many times as possible, and then press + until you reach $n$.
timblack 2023-05-23 19:09:27
A method like this is sometimes called a greedy strategy, because you are doing the thing that looks best in the moment, without checking to see if it's really what's best in the long run. Sometimes it really is the best, but often times it isn't.
MisakaMikasa 2023-05-23 19:09:41
no, it is bad if n is not close to a power of 2
bobjoebilly 2023-05-23 19:09:41
no, because if we have something like $2^n - 1$ it will be very inefficient
timblack 2023-05-23 19:09:49
No, this is not the best method; it does not result in the fewest number of button presses. What's an $n$ where this doesn't give the shortest sequence?
NL008 2023-05-23 19:10:23
127
winstonDeGreef 2023-05-23 19:10:23
63
AP0LL0-11 2023-05-23 19:10:23
12
Mahikohli 2023-05-23 19:10:23
15
timblack 2023-05-23 19:10:29
These are all good examples
timblack 2023-05-23 19:11:04
If we followed this method for $n = 15$, we would press + to get $1$, then we would press * three times to get $8$, then we couldn't press * anymore, so we would have to press + seven times to finish off: Z+***+++++++ $\rightarrow 15$. What's a shorter sequences of presses to get to $n = 15$?
NoahLeaf 2023-05-23 19:11:54
Z+*+*+*+
LongJumpMain 2023-05-23 19:11:54
Z+++*+*+
akmathisfun2020 2023-05-23 19:11:54
Z+*+*+*+
Chameleon07 2023-05-23 19:11:54
Z+*+*+*+
timblack 2023-05-23 19:11:55
Yep, Z+*+*+*+ $\rightarrow 15$, and that's only seven button presses instead of eleven.
timblack 2023-05-23 19:12:24
I want to point this out especially because a "greedy" strategy like this is tempting, but doesn't always work.
timblack 2023-05-23 19:12:35
Sometimes it works, but when it does, you need to PROVE it to be sure
timblack 2023-05-23 19:12:41
For now, we need to go back to the drawing board.
timblack 2023-05-23 19:13:13
I see some good suggestions in the chat already
timblack 2023-05-23 19:13:32
If you don't have ideas yet, for problems like this, it's a good idea to try some small values of $n$, and then look for a pattern. If you are solving this on your own, you might try all $n$ up to (say) $20$, and work out the shortest sequence for each.
timblack 2023-05-23 19:13:43
Here are shortest sequences for $n$ between $0$ and $20$:
timblack 2023-05-23 19:13:51
Z $\rightarrow 0$.
Z+ $\rightarrow 1$.
Z+* $\rightarrow 2$.
Z+*+ $\rightarrow 3$.
Z+** $\rightarrow 4$.
Z+**+ $\rightarrow 5$.
Z+*+* $\rightarrow 6$.
Z+*+*+ $\rightarrow 7$.
Z+*** $\rightarrow 8$.
Z+***+ $\rightarrow 9$.
Z+**+* $\rightarrow 10$.
Z+**+*+ $\rightarrow 11$.
Z+*+** $\rightarrow 12$.
Z+*+**+ $\rightarrow 13$.
Z+*+*+* $\rightarrow 14$.
Z+*+*+*+ $\rightarrow 15$.
Z+**** $\rightarrow 16$.
Z+****+ $\rightarrow 17$.
Z+***+* $\rightarrow 18$.
Z+***+*+ $\rightarrow 19$.
Z+**+** $\rightarrow 20$.
timblack 2023-05-23 19:14:02
Do you notice any patterns?
winstonDeGreef 2023-05-23 19:14:35
there are no consecutive pluses
TheBeast5520 2023-05-23 19:14:35
Never two +s in a row
vrondoS 2023-05-23 19:14:35
no consecutive +s
timblack 2023-05-23 19:15:04
The operation + doesn't appear twice in a row. This is a good observation that holds at least for $n$ up to $20$. I'd like to investigate whether it always holds. Let's take a look at this for a minute, and then come back to other observations
timblack 2023-05-23 19:15:24
The sequence Z+***+**+++*+* is a fairly arbitrary sequence that happens to result in $158$. Can you find a shorter sequence that gives $158$? Ideally you can find this while doing almost no arithmetic.
TheBeast5520 2023-05-23 19:16:05
*++ can be condensed to +*
NoahLeaf 2023-05-23 19:16:05
Because having *++ in a sequence is the same as ending it with +*, but is less efficient.
timchen 2023-05-23 19:16:05
Remove the double +s and replace them with a + before the previous *, to get Z+***+*+*+*+*
impromptuA 2023-05-23 19:16:05
Z+***+*+*+*+*
WangJY01 2023-05-23 19:16:05
Z+***+*+*+*+*
timblack 2023-05-23 19:16:27
Yes, Z+***+*+*+*+*. Specifically, we've taken a *++ out of the original sequence, and replaced it with +*. This reduces the total number of button presses by one.
timblack 2023-05-23 19:16:37
We are always allowed to replace *++ with +*, because if $m$ is the number we have before those presses, we either do *++ to get $(m \times 2) + 1 + 1 = 2m + 2$, or we do +* to get $(m + 1) \times 2 = 2m + 2$. Either way we get the same value!
timblack 2023-05-23 19:16:54
If we have a shortest sequence from $0$ to $n$, then it can't have *++ in it, or else we could do the operation above, meaning it wasn't actually the shortest.
timblack 2023-05-23 19:17:08
Actually, that means that a shortest sequence can't have two or more + operations in a row, except possibly at the beginning of the sequence, because otherwise those + operations will be preceded by a *, so we would have a *++ in there.
timblack 2023-05-23 19:17:34
There is a shortest sequence without two or more + operations in a row at the beginning either, because we can always replace Z++ with Z+*; either way you get $2$.
bobjoebilly 2023-05-23 19:17:41
beginning it doesn't matter since Z++ is the same as Z+*
timblack 2023-05-23 19:17:57
So, we can now say conclusively that there is a shortest sequence without two or more + operations in a row.
timblack 2023-05-23 19:18:26
Let's find some more patterns. Here's the first 20 again:
timblack 2023-05-23 19:18:27
Z $\rightarrow 0$.
Z+ $\rightarrow 1$.
Z+* $\rightarrow 2$.
Z+*+ $\rightarrow 3$.
Z+** $\rightarrow 4$.
Z+**+ $\rightarrow 5$.
Z+*+* $\rightarrow 6$.
Z+*+*+ $\rightarrow 7$.
Z+*** $\rightarrow 8$.
Z+***+ $\rightarrow 9$.
Z+**+* $\rightarrow 10$.
Z+**+*+ $\rightarrow 11$.
Z+*+** $\rightarrow 12$.
Z+*+**+ $\rightarrow 13$.
Z+*+*+* $\rightarrow 14$.
Z+*+*+*+ $\rightarrow 15$.
Z+**** $\rightarrow 16$.
Z+****+ $\rightarrow 17$.
Z+***+* $\rightarrow 18$.
Z+***+*+ $\rightarrow 19$.
Z+**+** $\rightarrow 20$.
timblack 2023-05-23 19:18:33
Anything else stand out?
floradaisy136 2023-05-23 19:18:58
odd ones end with +
angie. 2023-05-23 19:19:03
Odd numbers always end with a plus.
JigglypuffDaBest 2023-05-23 19:19:10
odd number always ends in +?
timblack 2023-05-23 19:19:19
For odd $n$, the sequence ends in +. Even though we're just noticing this as a pattern, we can actually prove that to get to an odd number, we have to end with +. That's because if we ended with * instead, we would have a whole number times two which is (the definition of) even.
epiconan 2023-05-23 19:19:57
oh even number also end with *
timblack 2023-05-23 19:20:05
For even $n$, the sequence ends in *. This sort of make sense: For example, to get to $n = 20$, we could end with a + by doing $19 + 1 = 20$ or we could end with a * by doing $10 \times 2 = 20$. Of those, going from $10$ to $20$ is a much bigger "jump" than $19$ to $20$.
EpicBird08 2023-05-23 19:20:28
Let's work backward!
mockingjay11 2023-05-23 19:20:28
Try the reverse method
akmathisfun2020 2023-05-23 19:20:28
Would it work if I solved the problem backwards, with the buttons $-1$ and $\div 2$, and tried to get from $n$ to $0$?
MisakaMikasa 2023-05-23 19:20:28
Work backwards
timblack 2023-05-23 19:20:38
I like this idea of thinking backwards to try to get from $n$ back to $0$. Let's carefully write down our method for constructing the sequence of button presses:
LongJumpMain 2023-05-23 19:21:03
The last operation to get to an even number is * and the last to get to an odd number is +
winstonDeGreef 2023-05-23 19:21:03
now you can work back from n, because if n is even, the last operation must be *, and if its odd the last operation must be +
TheBeast5520 2023-05-23 19:21:03
Recursive. For an odd n, it is shortest sequence for n-1 plus a +. For an even n it is shortest sequence for n/2 plus a *
timblack 2023-05-23 19:21:11
Start by writing down $n$ on a piece of paper. We build the sequence backwards. If the number is odd, choose +, then scratch out the number and replace it with $n - 1$. If instead the number is even, choose *, then scratch out the number and replace it with $n / 2$. Repeat until you get down to zero, then stop. Remember that we have chosen the button presses in reverse order.
timblack 2023-05-23 19:21:32
Let's do an example of following this procedure. Let's arbitrarily take $n = 42$.
timblack 2023-05-23 19:21:36
So, $42$ is the answer, but what is the question sequence? Follow this method and see if you can come up with the sequence of presses. Z?????? $\rightarrow 42$.
Curious_Droid 2023-05-23 19:22:14
42 -> 21 -> 20 -> 10 -> 5 -> 4 -> 2 -> 1
Curious_Droid 2023-05-23 19:22:14
Z+**+**+*
Technodoggo 2023-05-23 19:22:17
Z+**+**+*
FructosePear 2023-05-23 19:22:17
+**+**+*
timblack 2023-05-23 19:22:30
$42$ is even. We choose *, then replace $42$ with $42 / 2 = 21$.
timblack 2023-05-23 19:22:37
$21$ is odd. We choose +, then replace $21$ with $21 - 1 = 20$.
timblack 2023-05-23 19:22:43
$20$ is even. We choose *, then replace $20$ with $20 / 2 = 10$.
timblack 2023-05-23 19:22:47
$10$ is even. We choose *, then replace $10$ with $10 / 2 = 5$.
timblack 2023-05-23 19:22:52
$5$ is odd. We choose +, then replace $5$ with $5 - 1 = 4$.
timblack 2023-05-23 19:22:57
$4$ is even. We choose *, then replace $4$ with $4 / 2 = 2$.
timblack 2023-05-23 19:23:01
$2$ is even. We choose *, then replace $2$ with $2 / 2 = 1$.
timblack 2023-05-23 19:23:06
$1$ is odd. We choose +, then replace $1$ with $1 - 1 = 0$.
timblack 2023-05-23 19:23:17
$0$ is our stopping condition, so we are done.
timblack 2023-05-23 19:23:31
Remember that we need to read off our button presses in reverse order, so the sequence of presses is Z+**+**+* $\rightarrow 42$.
MisakaMikasa 2023-05-23 19:23:42
How do we rigorously prove that working backwards always results in the most efficient sequence?
timblack 2023-05-23 19:23:48
This will be our method for constructing the sequence. But we are not done. Remember that the instructions say, "As with all problems on the Qualifying Quiz, make sure to justify your answer with a proof." This is the most important part!
timblack 2023-05-23 19:24:11
Here's what we need to prove:
1. The sequence really results in $n$.
2. When $n$ is odd, the last button press in a shortest sequence is +.
3. When $n$ is even, the last button press in a shortest sequence is *.
We'll take these one at a time.
timblack 2023-05-23 19:24:26
1. The sequence really results in $n$.
Notice that when we cross out $a$ and replace it with $b$, we choose the operation that goes from $b$ back to $a$. So hopefully you can see why following the button presses backwards buulds from $0$ up to $n$.
timblack 2023-05-23 19:24:48
If you want to be formal in your proof, you can use induction on the number of button presses to show that the the number written down at any given time is the number that results from the remaining button presses in reverse order. But for the sake of moving along, I'm not going to do that induction here.
timblack 2023-05-23 19:25:13
2. When $n$ is odd, the last button press in a shortest sequence is +.
We addressed this one before. To get an odd number, you must end in +, because if you ended in * you'd have an even number.
timblack 2023-05-23 19:25:44
3. When $n$ is even, the last button press in a shortest sequence is *.
This is the hardest statement to prove, but we've already done the hard part. For the sake of contradiction, if the last button press had to be +, what would the second-to-last button press have to be?
NoahLeaf 2023-05-23 19:26:11
3. Otherwise, the sequence would have to end in *++, which can be condensed to +*
Shreyasharma 2023-05-23 19:26:11
+
epiconan 2023-05-23 19:26:11
+
miguel00 2023-05-23 19:26:11
It would have to be +
timblack 2023-05-23 19:26:21
Right, before the last +, we would have an odd number, which means before that we would have had to have another +. Do you see the problem?
timchen 2023-05-23 19:26:47
We said that was bad.
oyc_1975 2023-05-23 19:26:47
*++ --> +*
winstonDeGreef 2023-05-23 19:26:47
the ++ can be condensed to + before a *
timblack 2023-05-23 19:26:49
We showed earlier that there is a shortest sequence without two + operations in a row. So, we have a contradiction; the last button press doesn't have to be +; it can be *.
timblack 2023-05-23 19:27:07
This finishes the proof!
timblack 2023-05-23 19:27:45
The problem doesn't actually ask for the length of a shortest sequence of presses, just the sequence itself. But if you want to describe the number of presses, you can describe it as
NL008 2023-05-23 19:27:47
It is binary, + is 1 and * is 0
NL008 2023-05-23 19:27:47
It kind of resembles binary
timblack 2023-05-23 19:28:09
You can describe it as (length of binary representation of $n$) + (number of 1s in the binary representation of $n$) - 1. I'm not going to prove that today, but it's very related to the sequence of presses itself.
winstonDeGreef 2023-05-23 19:28:15
*+ is one and * is 0
timblack 2023-05-23 19:28:33
That was a lot of work, but remember that for this first problem, I wanted to go through not just the sequence and the proof, but the whole problem solving process as well. :)
timblack 2023-05-23 19:28:38
Remember that you are welcome to ask questions in the chat at any point. I'm going to take just another minute before moving on to problem 2.
ninjatutle 2023-05-23 19:29:19
is it really a funny calculator though
Technodoggo 2023-05-23 19:29:35
it doesn't sound like a calculator...
timblack 2023-05-23 19:30:38
Okay! Problem 2!
timblack 2023-05-23 19:30:55
2. Ordinarily, when an object bounces off of a surface — whether it's light reflecting from a mirror or a billiard ball bouncing off the side of a billiards table — its path makes the same angle with the surface before and after the bounce. However, a Bizarro Billiards table behaves differently.



The table is a rectangle with two horizontal and two vertical sides in the $x$--$y$ plane. The rule that determines how balls bounce is:



- If the ball is moving up and right along a line with slope $1$, and it hits the top side of the table, it bounces off and continues moving down and right along a line with slope $-1/2$.

- If the ball is moving up and right along a line with slope $2$, and it hits the top side of the table, it bounces off and continues moving down and right along a line with slope $-1$.

- These two bounces are reversible: if the ball is moving up and left along a line with slope $-1/2$ or $-1$, it bounces off and continues moving down and left along a line with slope $1$ or slope $2$, respectively.

- When the ball is bouncing off another side of the table, the rule for bouncing is the same as it would be if you rotated the table to make that side the top side.

\end{itemize}



This rule is summarized in the diagram below.
timblack 2023-05-23 19:31:02
https://mathcamp.org/files/yearly/2023/quiz/bizarro-billiards.jpg
timblack 2023-05-23 19:31:52
There were three parts to the problem. We'll take them one at a time.
timblack 2023-05-23 19:31:55
(a) Suppose that the ball starts in the top left corner (point $B$) moving down and right along a line with slope $-1$. If the ball hits side $AD$ and bounces off, then hits side $BC$ and bounces off, and then ends up at corner $D$, what must the proportions of the rectangle be?
timblack 2023-05-23 19:32:11
The trajectory of the ball in this problem is shown in the diagram below.
timblack 2023-05-23 19:32:15
https://mathcamp.org/files/yearly/2023/math_jam/qq2sol1.png
timblack 2023-05-23 19:32:31
On the downward bounces, the ball moves with slope $-1$
timblack 2023-05-23 19:32:42
On the upward bounce, the ball moves with slope $2$
timblack 2023-05-23 19:33:25
So, if $AB = 1$, how far apart are the bounce points?
oyc_1975 2023-05-23 19:33:41
so if $AB = 1$ then we can cut up the path by slope and have $AD = 1+0.5+1 = 2.5$
oyc_1975 2023-05-23 19:33:41
$1,0.5,1$
timblack 2023-05-23 19:33:52
On the downward bounces, the ball moves with slope $-1$; therefore, every time it moves a vertical distance of $AB$, it moves a horizontal distance of $AB$ as well.
timblack 2023-05-23 19:34:04
On the upward bounce, the ball moves with slope $2$; therefore, when it moves a vertical distance of $AB$, it only moves a horizontal distance of $\frac12AB$.
timblack 2023-05-23 19:34:15
Its total horizontal distance traveled is $AB + \frac12AB + AB = \frac52 AB$; therefore, if it ends up traveling a distance of $BC$ horizontally, the proportions of the rectangle must be $AB : BC = 1: \frac52 = 2:5$.
timblack 2023-05-23 19:34:21
That's (a)!
Shreyasharma 2023-05-23 19:34:43
:)
timblack 2023-05-23 19:34:52
(b) Suppose that the ball starts in the bottom left corner (point $A$) moving up and right along a line with slope $1$. If the ball hits side $BC$ and bounces off, then hits side $CD$ and bounces off, then hits side $AD$ and bounces off, and then ends up at corner $B$, what must the proportions of the rectangle be?
timblack 2023-05-23 19:35:01
The trajectory of the ball in this problem is shown in the diagram below. Let $X$, $Y$, and $Z$ be the locations of the three points where the ball bounces (on sides $BC$, $CD$, and $AD$, respectively).
timblack 2023-05-23 19:35:05
https://mathcamp.org/files/yearly/2023/math_jam/qq2sol2.png
timblack 2023-05-23 19:35:19
We can do similar accounting here, with a bit more work
timblack 2023-05-23 19:35:51
We should trace through the slopes:
timblack 2023-05-23 19:35:54
BZ?
Curious_Droid 2023-05-23 19:36:11
-1/2
JigglypuffDaBest 2023-05-23 19:36:11
-1/2
timblack 2023-05-23 19:36:13
Slope $-1/2$
timblack 2023-05-23 19:36:17
ZY?
JigglypuffDaBest 2023-05-23 19:36:29
1
happypi31415 2023-05-23 19:36:29
$1$
vrondoS 2023-05-23 19:36:29
1
spm269 2023-05-23 19:36:29
1?
timblack 2023-05-23 19:36:34
YX?
HappyBunny 2023-05-23 19:36:47
-1/2
GryFry 2023-05-23 19:36:47
-1/2
timblack 2023-05-23 19:36:51
$-1/2$
timblack 2023-05-23 19:36:58
And XA?
Chameleon07 2023-05-23 19:37:04
1
CrunchyCucumber 2023-05-23 19:37:04
1
timblack 2023-05-23 19:37:08
$1$.
timblack 2023-05-23 19:37:20
We know that $AX$ and $YZ$ are parallel lines with slope $1$ (though the ball moves in opposite directions along those lines). Therefore $BX = AB$ and $DY = DZ$.
timblack 2023-05-23 19:37:31
Meanwhile, $XY$ and $ZB$ are parallel lines with slope $-\frac12$ (though, again, the ball moves in opposite directions along those lines). Therefore $XC = 2 CY$ and $AZ = 2AB$.
timblack 2023-05-23 19:37:42
Because only the proportions of the rectangle matter, let $AB = 1$, so that $BX = 1$ and $AZ = 2$.
timblack 2023-05-23 19:37:56
We do not yet know $XC$, so let $XC = x$; then $CY = \frac12 x$, $DY = 1 - CY = 1 - \frac12x$, and $DZ = DY = 1 - \frac12x$.
timblack 2023-05-23 19:38:20
Because $BC = AD$, we get an equation to solve for $x$:



$BX + XC = AZ + ZD \implies 1 + x = 2 + (1 - \frac12x).$
timblack 2023-05-23 19:38:37
Solve for $x$?
miguel00 2023-05-23 19:39:31
x=4/3
impromptuA 2023-05-23 19:39:31
$x=\frac{4}{3}$
NoahLeaf 2023-05-23 19:39:31
x=4/3
AoPSyang 2023-05-23 19:39:31
4/3
timblack 2023-05-23 19:39:33
Solving, we get $x = \frac43$.
timblack 2023-05-23 19:39:40
So $BC = \frac73$: the proportions of the rectangle must be $AB : BC = 1 : \frac73 = 3 : 7$.
timblack 2023-05-23 19:39:52
And that's (b) :O
rithek.shankar 2023-05-23 19:40:01
Yay
Mahikohli 2023-05-23 19:40:12
so I did my QT through coordinates, should I have done it this way?
JigglypuffDaBest 2023-05-23 19:40:12
is it ok if I did this in a much more complicated way with much more confusing variables
timblack 2023-05-23 19:40:21
Yes, there are lots of different ways to solve this
timblack 2023-05-23 19:40:35
For that matter, there are usually multiple ways to solve all the problems on the Qualifying Quiz
timblack 2023-05-23 19:40:39
It's more interesting that way :)
JigglypuffDaBest 2023-05-23 19:41:02
(i know clarity is a big thing)
timblack 2023-05-23 19:41:22
Clarity is always good. But lots of other things are good too. :)
timblack 2023-05-23 19:41:34
(c) Suppose that the rectangle has height $AB = 3$ and width $BC = 5$, and the ball starts in the bottom left corner (point $A$) moving up and right along a line with slope $1$. Describe the trajectory that the ball takes.
timblack 2023-05-23 19:41:52
The first few bounces of the ball are shown in the first diagram below.
timblack 2023-05-23 19:41:57
https://mathcamp.org/files/yearly/2023/math_jam/qq2sol3.png
timblack 2023-05-23 19:42:12
A few calculations quickly tell us that we're not going to end up in a corner like last time; the collision points have denominators that, experimentally, are increasing powers of $2$.
Curious_Droid 2023-05-23 19:42:29
it kind of goes to the path where the ball keeps returning to its original position after four bounces
miguel00 2023-05-23 19:42:39
The ball slowly approaches a tradjectory
timblack 2023-05-23 19:42:59
I like these descriptions. In fact, where the ball "wants" to be is the cyclic trajectory shown in the second diagram below: repeatedly visiting the two points at a distance $1$ from $B$ (along $AB$ and $BC$) and the two points at a distance $1$ from $D$ (along $CD$ and $AD$). We can check that if $AB = CD = 3$ and $BC = AD = 5$, then the sides of this cyclic trajectory have the correct slopes. Unfortunately, the only way to end up on this cyclic trajectory is to start on it; the ball we actually have will never get there.
timblack 2023-05-23 19:43:08
https://mathcamp.org/files/yearly/2023/math_jam/qq2sol4.png
oyc_1975 2023-05-23 19:43:17
slowly approaches a stable loop but never really reaches it
timblack 2023-05-23 19:43:27
One way to describe the actual path the ball takes is to keep track of the following distances:

- When the ball bounces off $AB$ or $BC$, how far away is it from $B$?

- When the ball bounces off $CD$ or $AD$, how far away is it from $D$?
timblack 2023-05-23 19:43:38
We can begin by computing these distances for the first few bounces, to see a pattern.
timblack 2023-05-23 19:43:51
The ball starts at point $A$, which at distance $3$ from $B$ along $AB$.
timblack 2023-05-23 19:44:02
The next bounce is on $BC$, at distance ??? from $B$.
Curious_Droid 2023-05-23 19:44:42
3
DU4532 2023-05-23 19:44:42
also $3$
oyc_1975 2023-05-23 19:44:42
3
JigglypuffDaBest 2023-05-23 19:44:42
3
timblack 2023-05-23 19:44:45
The next bounce is on $BC$, also at distance $3$ from $B$.
timblack 2023-05-23 19:45:02
The next bounce is on $CD$. How far from $D$?
AoPSyang 2023-05-23 19:45:41
2
oyc_1975 2023-05-23 19:45:41
2
NoahLeaf 2023-05-23 19:45:41
2
timchen 2023-05-23 19:45:41
Wait $2$
timblack 2023-05-23 19:45:44
Traveling $2$ units horizontally, the ball travels $1$ unit vertically, so it hits $CD$ at distance $2$ from $D$.
timblack 2023-05-23 19:46:07
The next bounce is on $AD$, also at distance $2$ from $D$ (since we're traveling along a slope of $1$).
timblack 2023-05-23 19:46:17
The next bounce is on $AB$. Traveling $3$ units horizontally, the ball travels $\frac32$ units vertically, so it hits $CD$ at distance $\frac32$ from $B$.
timblack 2023-05-23 19:46:33
We've now bounced off each wall once.
timblack 2023-05-23 19:46:51
The next bounce is on $BC$, at what distance from $B$?
Chameleon07 2023-05-23 19:47:39
3/2
rithek.shankar 2023-05-23 19:47:39
3/2
impromptuA 2023-05-23 19:47:39
3/2
Mahikohli 2023-05-23 19:47:39
3/2
timblack 2023-05-23 19:47:41
Right, we're again traveling along a path of slope 1, so we get an isosceles right triangle. The next bounce is on $BC$, also at distance $\frac32$ from $B$.
timblack 2023-05-23 19:47:57
The next bounce is on $CD$. Traveling $5 - \frac32 = \frac72$ units horizontally, the ball travels $\frac74$ units vertically, so it hits $CD$ at distance $3 - \frac74 = \frac54$ from $D$.
timblack 2023-05-23 19:48:21
(We're again traveling along a path of slope $-1/2$)
timblack 2023-05-23 19:48:30
Knowing that the first few distances we measure are $3$, $2$, $\frac32$, $\frac54$ and that ultimately they should be getting closer and closer to $1$, we can conjecture the following rule:
timblack 2023-05-23 19:48:43
Conjecture. If we look at $4$ consecutive bounces of the ball, off sides $AB$, $BC$, $CD$, $AD$ in that order, the first two bounces are at distance $1 + (\frac12)^{2k-1}$ from $B$, and the last two bounces are at distance $1 + (\frac12)^{2k}$ from $D$. The value of $k$ increases with each iteration.
miguel00 2023-05-23 19:49:06
We can prove this by induction
timblack 2023-05-23 19:49:16
Good idea! Once we have conjectured this rule, we can prove it by induction. The first two points where the ball touches the rectangle are indeed at distance $3 = 1 + (\frac12)^{-1}$ from $B$, and the next two are at distance $2 = 1 + (\frac12)^0$ from $D$, which is what the conjecture predicts for $k=0$.
timblack 2023-05-23 19:49:32
Suppose that one cycle of $4$ bounces happened as predicted. Then, leaving side $AD$ moving up and left along a line of slope $-\frac12$, the ball has $5 - (1 + (\frac12)^{2k}) = 4 - (\frac12)^{2k}$ to travel horizontally, so it travels $2 - (\frac12)^{2k+1}$ vertically, and hits side $AB$ at distance $3 - (2 - (\frac12)^{2k+1}) = 1 + (\frac12)^{2k+1}$ from $B$, as predicted.
timblack 2023-05-23 19:49:52
After that, the ball bounces along a line with slope $1$, so it hits $BC$ at the same distance $1 + (\frac12)^{2k+1}$ from $B$.
timblack 2023-05-23 19:50:12
After that, the ball bounces along a line with slope $-\frac12$ and has a horizontal distance of $5 - (1 + (\frac12)^{2k+1}) = 4 - (\frac12)^{2k+1}$ to travel until it hits $CD$. It travels a vertical distance of $2 - (\frac12)^{2k+2}$ in the same time, and hits $CD$ at a distance of $3 - (2 - (\frac12)^{2k+2}) = 1 + (\frac12)^{2k+2}$ from $D$.
timblack 2023-05-23 19:50:30
Ooof, a lot of formulas
timblack 2023-05-23 19:50:43
But the nice thing about formulas is that you can sit down and grind them out
timblack 2023-05-23 19:50:52
Almost done.
timblack 2023-05-23 19:50:54
After that, the ball bounces along a line with slope $1$, so it hits $AD$ at the same distance of $1 + (\frac12)^{2k+2}$ from $D$.
timblack 2023-05-23 19:51:20
That's right where we are supposed to be after one cycle!
timblack 2023-05-23 19:51:26
We have completed one more iteration of the cycle in accordance with the conjecture. By induction on $k$, we will continue bouncing in the same way forever.
timblack 2023-05-23 19:52:03
That's wraps up problem 2!
timblack 2023-05-23 19:52:21
I'll pause for a moment before moving on to problem 3
timblack 2023-05-23 19:52:40
This is a good time to use the chat for questions, or to express your sense of excitement and adventure
EpicBird08 2023-05-23 19:52:45
problem 3b was WONDERFUL
Curious_Droid 2023-05-23 19:53:16
a pool table couldnt realistically have this property... right?
timblack 2023-05-23 19:53:20
I don't think so :(
timblack 2023-05-23 19:53:28
I thought about it a bit...
rithek.shankar 2023-05-23 19:54:00
Magnetic ones chould
timblack 2023-05-23 19:54:03
hmmm
rithek.shankar 2023-05-23 19:54:09
Time for 3
rithek.shankar 2023-05-23 19:54:09
3 is my favorite
timblack 2023-05-23 19:54:15
Okay, let's do it
timblack 2023-05-23 19:54:55
3. You have $4046$ identical-looking coins, but $2023$ of them weigh $1$ gram each, while the other $2023$ coins weigh $2$ grams each. You cannot distinguish these coins in any way manually. However, you have a partially-working scale, tuned to some unknown integer value $X$: when you weigh a set of coins, the scale reports if the weight is ``less than $X$ grams'' or ``at least $X$ grams''. You know that $1 < X < 6070$.
timblack 2023-05-23 19:55:16
One of my favorite problems too. I read a lot of submissions, so thought a lot about it :O
timblack 2023-05-23 19:55:24
(a) Prove that you can eventually find a set of coins that weighs exactly X grams.
timblack 2023-05-23 19:55:32
(b) Prove that you can find such a set in at most 10000 weighings.
timblack 2023-05-23 19:55:46
Note the scale never tell you "exactly $X$ grams." If the coins on the scale weigh exactly $X$ grams, the scale will still just tell you "at least $X$ grams."
Curious_Droid 2023-05-23 19:56:09
i am still finding bugs in my submission to 3b to this day
timblack 2023-05-23 19:56:14
Thank you for sharing this!
timblack 2023-05-23 19:56:20
I think it captures a common sentiment
timblack 2023-05-23 19:56:26
There are a lot of moving parts here
timblack 2023-05-23 19:56:35
Technically, if you solve part (b), you've automatically solved part (a). But this problem has a lot of moving parts, and it's especially easy to overlook something in part (b). So, if you can come up with a separate, simpler solution that works just for part (a) that you are sure is right, it was probably a good idea to write that up separately and submit it. So, we'll come up with a solution to (a) first.
timblack 2023-05-23 19:57:18
Actually, we'll start with a bit of a side observation (but keep in mind there are many different ways to approach this)
timblack 2023-05-23 19:57:20
Let set $S$ and $T$ be two equally sized sets of coin that differ by a single coin. That is, you can get from $S$ to $T$ by removing one coin and adding a different one. Suppose that you weigh each set on the broken scale, and find that $S$ has weight "less than $X$ grams" and $T$ has weight "at least $X$ grams." What more specifically can you say about the weight of $S$ and $T$?
lpieleanu 2023-05-23 19:58:53
$S = X-1, T = X$
Curious_Droid 2023-05-23 19:58:53
S must be X-1 and T must be X?
timchen 2023-05-23 19:58:53
$T=X,S=X-1$
akmathisfun2020 2023-05-23 19:58:53
T is X grams and S is X-1 grams
CrunchyCucumber 2023-05-23 19:58:53
T weighs X and S weights X-1
timblack 2023-05-23 19:58:58
I'd prefer to say that $T$ weighs $X$ grams rather than $T = X$, but otherwise I agree.
timblack 2023-05-23 19:59:08
$S$ weighs exactly $X - 1$ grams, and $T$ weighs exactly $X$ grams.
timblack 2023-05-23 19:59:24
Exactly! "Exactly" is hard to come by in this problem
timblack 2023-05-23 19:59:30
All the coins weigh either $1$ or $2$ grams, so when you swap out one coin for another, it can change the weight of the pile by at most $1$ gram. Since $S$ weighs at most $X - 1$ grams and $T$ weighs at least $X$ grams, and the sets can differ by at most $1$ gram, we know exactly how much each of those two piles weigh.
timblack 2023-05-23 19:59:52
Now let's say $S$ and $T$ are two sets with the same size, but now these sets might have any number of coins in common, or none at all. Again, say that you weigh each set, and find $S$ weights "less than $X$ grams" while $T$ weighs "at least $X$ grams." How can you use the broken scale to find a set that weighs exactly $X$ grams?
oyc_1975 2023-05-23 20:00:44
keep on swapping coins?
Curious_Droid 2023-05-23 20:00:44
try swapping coins between sets
timblack 2023-05-23 20:00:50
Something to do with swapping coins...
CrunchyCucumber 2023-05-23 20:01:02
Transfer coins between the two sets?
NoahLeaf 2023-05-23 20:01:11
Switch coins from one to the other one at a time
timblack 2023-05-23 20:01:29
Start with $S$ on the broken scale. We'll swap out coins from $S$ one at a time for coins from $T$, and after each swap we'll take a weighing.
timblack 2023-05-23 20:01:40
If you prefer set notation, here's a more precise way to say this:

- Let $V$ be the intersection of $S$ and $T$.

- Let $s_1, s_2, \dots, s_k$ be the coins that are in $S$ but not $T$.

- Let $t_1, t_2, \dots, t_k$ be the coins that are in $T$ but not $S$.

- So $S = V \cup \{s_1, s_2, \dots, s_k\}$ and $T = V \cup \{t_1, t_2, \dots, t_k\}$.

- For $i = 0, 1, \dots, k$, let $S_i = V \cup \{t_1, t_2, \dots, t_i, s_{i + 1}, s_{i + 2}, \dots, s_k\}$.

- So, $S_0 = S$ and $S_k = T$.

- Each $S_i$ differs from $S_{i + 1}$ by swapping out exactly one coin.

- We weigh $S_0 = S$, then $S_1$, then $S_2$, and so on on the broken scale.
timblack 2023-05-23 20:02:45
But frankly, the set notation doesn't matter too much here if you understand what I mean by swapping coins one at a time
timblack 2023-05-23 20:03:00
We already know weighing $S$ results in "less than $X$ grams." Then we swap out a coin, and we might get "less than $X$ grams," or we might get "at least $X$ grams." If we get "at least", stop. As long as we keep getting "less than," swap out another coin and weigh again. We know we'll eventually see "at least," because if nothing stops us earlier, we'll eventually have $T$ on the scale, which we know weighs "at least $X$ grams."
GryFry 2023-05-23 20:03:18
when set s becomes at least X grams, S weighs exactly X grams
timblack 2023-05-23 20:03:53
Nice. Let's think about that first set $S_i$ that weighs "at least $X$ grams." We know that we got that set by taking another set, $S_{i - 1}$ that weighed "less than $X$ grams," and swapping a single coin. So $S_i$ must weigh exactly $X$ grams.
timblack 2023-05-23 20:04:14
Since we already knew the weight of $S_0 = S$ and $S_k = T$, this process takes at most $k - 1$ additional weighings, where $k$ is at most the size of $S$ (which equals the size of $T$).
timblack 2023-05-23 20:04:34
For a set $S$ of coins, let $\#S$ be the number of coins in $S$. To summarize:

Lemma: Let $S$ and $T$ be sets of coins with $\#S = \#T$. Suppose that $S$ weighs "less than $X$ grams" and $T$ weighs "at least $X$ grams." Then it is possible to find a set weighing exactly $X$ grams using fewer than $\#S$ additional weighings with the broken scale.
Curious_Droid 2023-05-23 20:04:54
well then we could be basically done as long as we find two same-size sets of coins one <X and one >=X?
timblack 2023-05-23 20:05:03
Yes, we now have one avenue to solving the problem!
timblack 2023-05-23 20:05:40
If we can find a pair of sets where one weighs less than $X$ grams and the other weighs at least, and the have the same number of coins, we can apply the lemma and be done
timblack 2023-05-23 20:05:57
Okay, so let's see if this helps with (a)
timblack 2023-05-23 20:06:01
In part (a), we're allowed an unlimited number of weighings with the broken scale. So if we want, we can just weigh every possible set of coins. How many weighings would that be?
happypi31415 2023-05-23 20:07:17
2^4046
wilsonstmartin 2023-05-23 20:07:17
$2^2024
NoahLeaf 2023-05-23 20:07:17
$2^{4046}$, although we can reduce this by one by not considering the empty set
timblack 2023-05-23 20:07:25
There are $4046$ coins, so there are $2^{4046}$ sets of coins. It would take... a while... to weigh them all, but it's allowed.
timblack 2023-05-23 20:07:43
If we do all those weighings, and we find two sets that have the same size, but where one weighs "less than $X$ grams" and the other weighs "at least $X$ grams," then that's great; we can apply the lemma to find a set that weighs exactly $X$ grams.
MisakaMikasa 2023-05-23 20:07:52
Is it always possible to find two sets that complies the conditions?
timblack 2023-05-23 20:07:54
Do you think there are always two such sets?
CrunchyCucumber 2023-05-23 20:08:55
yes
vrondoS 2023-05-23 20:08:55
yes
ninjatutle 2023-05-23 20:08:55
not always...
snow52 2023-05-23 20:08:55
no
lpieleanu 2023-05-23 20:08:55
yep
akmathisfun2020 2023-05-23 20:08:55
no
timblack 2023-05-23 20:08:59
Yes! Well... technically, no... If $X = 6069$, then only the set of all coins weighs "at least $X$ grams," and all other sets weigh "less than $X$ grams." But this case is easy to spot if you weigh all $2^{4046}$ sets, and in this case the answer is the set of all coins.
timblack 2023-05-23 20:09:17
But otherwise, as long as $X \neq 6069$, then yes, there do exist $S$ and $T$ with the same number of coins with $S$ weighing "less than $X$ grams" and $T$ weighing "at least $X$ grams." Here's why:
timblack 2023-05-23 20:09:36
Consider taking as many 2g coins as you can without going over $X$ grams (using all $2023$ 2g coins if necessary), and then adding to that as many 1g coins as you need to hit exactly $X$ grams. Let this set be $T$. Then, take away a 2g coin (there will be at least one 2g coin because $X \geq 2$) and replace it with a 1g coin (there will be at least one 1g coin left over because $X < 6069$). Let this set set be $S$; it weighs $X - 1$ grams.
timblack 2023-05-23 20:10:09
Since we don't know $X$, and we don't know which coins are 1g coins, and we don't know which coins are 2g coins, we cannot actually construct this specific $S$ and $T$, but all we need to know is that such an $S$ and $T$ exist. We weigh all $2^{4046}$ sets, find an appropriate $S$ and $T$ from that, then apply the lemma.
Curious_Droid 2023-05-23 20:10:34
epic
timblack 2023-05-23 20:10:37
:O
timblack 2023-05-23 20:10:44
That finishes part (a).
timblack 2023-05-23 20:11:20
Although, again, it's not the only way to do part (a).
winstonDeGreef 2023-05-23 20:11:23
thats completely different to how I solved a
timblack 2023-05-23 20:11:41
Now let's move onto part (b).
timblack 2023-05-23 20:11:49
(b) Prove that you can find such a set in at most 10000 weighings.
timblack 2023-05-23 20:12:03
We're not going to go for the absolute minimum number of weighings right now, though I'll make some comments on that at the end. We'll be content just to get the number under 10000. Which, incidentally, is a lot less than $2^{4046}$, so we have some work to do.
timblack 2023-05-23 20:12:29
We will start with an empty scale. Then, we'll add coins to the scale one at a time. Each time we add a coin, we'll take a reading. As long as the scale keeps saying "less than $X$ grams," we'll keep going. The first time the scale reads "at least $X$ grams," we'll stop. Let $P$ be the set of coins on the scale when we reach this stopping point.
timblack 2023-05-23 20:12:48
Since we were adding coins one by one, and set $P$ was the first set that weighed "at least $X$ grams," can we say that $P$ weighs exactly $X$ grams?
GryFry 2023-05-23 20:13:23
no:(
winstonDeGreef 2023-05-23 20:13:23
no, it might weigh X+1
TheBeast5520 2023-05-23 20:13:23
No, either X or X+1
MathFun1000 2023-05-23 20:13:23
no it could weight $X$ or $X+1$ grams
bburubburu 2023-05-23 20:13:23
no, it could be x+1
timblack 2023-05-23 20:13:53
That's right. No, we can't. Set $P$ might weigh exactly $X$ grams, but it might instead weigh $X + 1$ grams. That could happen if the scale previously weighed $X - 1$ grams (which reads as "less than $X$ grams") and then we add a 2g coin, resulting in $X + 1$ grams (which reads as "at least $X$ grams").
timblack 2023-05-23 20:14:04
The good news is that set $P$ can only read either $X$ grams or $X + 1$ grams. We know that it's at least $X$ because the scale told us so. And, we know that it's at most $X + 1$, because prior to the addition, the scale weighed at most $X - 1$ grams, and then we added one more coin weighing at most 2g, so the weight of the pile is at most $(X - 1) + 2 = X + 1$.
timblack 2023-05-23 20:14:29
From here, we're going to split into two cases:

- Case 1: $P$ consists of at least $2024$ coins.

- Case 2: $P$ consists of at most $2023$ coins.

We'll see why we're splitting it this way in a moment. We'll start with case 1.
timblack 2023-05-23 20:15:08
Getting this set $P$ that weighs $X$ or $X+1$ is key, but somehow it's still some work to narrow it down to just $X$.
timblack 2023-05-23 20:15:25
Case 1: $P$ consists of at least $2024$ coins.
timblack 2023-05-23 20:15:33
In this case, start with $P$ on the scale. Remove one coin from the scale, weigh the remainder, and then put the coin back so that you again have $P$ on the scale. Do this for every coin in $P$.
timblack 2023-05-23 20:15:37
Equivalently, in set notation, weigh $P \smallsetminus \{c\}$ for each coin $c$ in $P$.
timblack 2023-05-23 20:16:00
Now, since $P$ weighs either $X$ or just barely more than $X$, you would think that typically that $P$ minus one coin would weigh less than $X$ grams. However, not always. If $P$ minus some coin weighs "at least $X$ grams," what can you conclude?
vrondoS 2023-05-23 20:16:48
it is exactly X
akmathisfun2020 2023-05-23 20:16:48
we have X grams
zheng_ann 2023-05-23 20:16:48
those coins are 1g
Chameleon07 2023-05-23 20:16:48
You've hit X after removing coin
happypi31415 2023-05-23 20:16:48
the coin was worth 1
snow52 2023-05-23 20:16:48
it is exactly X grams
UnknownMonkey 2023-05-23 20:16:48
That coin you removed weighs 1 gram
AoPSyang 2023-05-23 20:16:48
it must equal to X grams
timblack 2023-05-23 20:16:54
Indeed. Since $P$ weighs at most $X + 1$ grams, and each coin weighs at least 1g, then $P$ minus a coin must weigh at most $(X + 1) - 1 = X$ grams. So if it also weighs at least $X$ grams, we can conclude that $P$ minus that coin weighs exactly $X$ grams.
timblack 2023-05-23 20:17:12
So, if there is any coin in $P$ for which $P$ minus that coin weighs "at least $X$ grams," then $P$ minus that coin weighs exactly $X$ grams, and we are done. Yay!

:)
DU4532 2023-05-23 20:17:21
yey
timblack 2023-05-23 20:17:33
That means that from now on we only need to worry about what happens when $P$ minus a coin weighs "less than $X$ grams" for every coin in $P$.
timblack 2023-05-23 20:18:06
But also, since $P$ has at least $2024$ coins, it must contain at least one 1g coin. Why?
shihan 2023-05-23 20:18:43
pigeonhole principle
wilsonstmartin 2023-05-23 20:18:43
only $2023$ 2 weight coins
ThousandthStar 2023-05-23 20:18:43
There are 2023 2g coins
cinnamoncrunch 2023-05-23 20:18:43
Because there are 2023 2g coins
Rinnypig 2023-05-23 20:18:43
Because there are only 2023 2g coins.
FlyingPig7 2023-05-23 20:18:43
because there are only 2023 2g coins
timblack 2023-05-23 20:18:55
There are only $2023$ 2g coins, so there just aren't enough of them for all of $P$ to be 2g coins.
timblack 2023-05-23 20:19:06
This, incidentally, is why I wanted to split off the case that $P$ consists of at least $2024$ coins.
timblack 2023-05-23 20:19:30
Okay, so $P$ has a 1g coin. Which means that $P$ minus a 1g coin weighs "less than X," So $P$ itself weighs less than $X + 1$ grams. But since $P$ weighs at least $X$ grams, that means it weighs exactly $X$ grams.
timblack 2023-05-23 20:20:03
So then we can conclude that $P$ itself weighs $X$ grams exactly, so again we have found our set of coins! :O
VanuTalin 2023-05-23 20:20:20
Horray!
miguel00 2023-05-23 20:20:20
:)
timblack 2023-05-23 20:20:23
So, that's how we find at set of weight exactly $X$ grams in case $1$. How many weighings does it take?
miguel00 2023-05-23 20:21:24
At most 4046*2
Curious_Droid 2023-05-23 20:21:24
4046 + 4046 = 8092?
shihan 2023-05-23 20:21:24
$\le 8092$ weighings
timblack 2023-05-23 20:21:27
Adding one coin at a time to the scale to find $P$ takes at most $4046$, and removing (and then replacing) each coin in turn takes another $4046$ at most, so we use at most $4046 + 4046 = 8092$ weighings in total.
timblack 2023-05-23 20:21:40
(I see lots of other numbers in the chat, which may be right if you are careful about it)
timblack 2023-05-23 20:21:54
That wraps up case 1. :D
ThousandthStar 2023-05-23 20:22:28
Why does replacing a coin count as a weighing?
timblack 2023-05-23 20:22:58
Replacing itself doesn't count as a weighing, but while the coin is removed we do a weighing
timblack 2023-05-23 20:23:20
Case 2: $P$ consists of at most $2023$ coins.
timblack 2023-05-23 20:23:31
Let $k = \#P$. That is, let $k$ be the number of coins in $P$.
MathFun1000 2023-05-23 20:24:01
In this case the complement of $P$ is equivalent of $P$ in case 1
timblack 2023-05-23 20:24:16
What do we think? Can we just do the same thing with the complement of the set?
timblack 2023-05-23 20:25:16
Well, maybe, but it would still take more work. This problem is not symmetric. For one, there is an asymmetry between "less than" and "at least" (note it's not "less than" and "greater than")
timblack 2023-05-23 20:25:31
And also, we don't get to "complement" $X$
timblack 2023-05-23 20:25:49
So, we'll really just have to do some more work :( Lots of people tried that though
timblack 2023-05-23 20:26:08
Let $k = \#P$. That is, let $k$ be the size of $P$.

If $k = 1$, then since $X \geq 2$, the only way to have $P$ weigh at least $X$ grams is to have $X = 2$, and $P$ consists of a single 2g coin. Then we can conclude that $P$ weighs exactly $X$, and we are set.
timblack 2023-05-23 20:26:21
So, from this point on, we can take $k \geq 2$.
timblack 2023-05-23 20:26:46
Lots of ways to proceed. Here's one of my favorite.
timblack 2023-05-23 20:26:47
Make a pile of all the coins not in $P$, and add to this pile up to $k - 1$ coins from $P$ so that the pile has a multiple of $k$ coins. Partition this pile into sets $Q_1, Q_2, \dots, Q_\ell$.
timblack 2023-05-23 20:27:12
Weigh each $Q_i$ on the broken scale. Let's say one of these $Q_i$ weighs "less than $X$ grams." What can we do?
shihan 2023-05-23 20:28:22
apply the lemma
GryFry 2023-05-23 20:28:22
part a strategy
lpieleanu 2023-05-23 20:28:22
switch out coins until it has at least $X$
timblack 2023-05-23 20:28:29
Ooo maybe our lemma applies.
timblack 2023-05-23 20:28:40
What are the sets we can use with the lemma?
timblack 2023-05-23 20:29:29
We need a set that weighs at least $X$ grams, and another set of the same size that weighs less than $X$ grams...
NoahLeaf 2023-05-23 20:29:46
We have to get $Q_i$ to have exactly k coins, not just a multiple
timblack 2023-05-23 20:30:06
Each $Q_i$ has size exactly $k$. Put together their size is a multiple of $k$.
Chameleon07 2023-05-23 20:30:32
P and the Qi?
ThousandthStar 2023-05-23 20:30:32
$S=Q_i$, $T=P$
Curious_Droid 2023-05-23 20:30:32
so just P and that Q_i
timblack 2023-05-23 20:30:52
Yes, nice! We have a pile $Q_i$ weighing "less than $X$ grams," and a pile $P$ with the same number of coins weighing "at least $X$ grams." So, we can find a set weighing exactly $X$ grams using our lemma with $S = Q_i$ and $T = P$!
Shivasaurus 2023-05-23 20:31:15
ohhhh
lrjr24 2023-05-23 20:31:17
wait why can't you go from X-1 to X+1 in this lemma and skip over a set that weighs exactly X grams
timblack 2023-05-23 20:32:07
If we just removed a coin, could change weight by 1g or 2g. But since we replace the coin, the difference between two coins is only at most 1g (if one is a 1g coin and the other is a 2g coin).
timblack 2023-05-23 20:32:16
So now there's only one remaining sticking point: We just need to handle the situation where every $Q_i$ weighs "at least $X$ grams."
timblack 2023-05-23 20:32:36
I claim that in this situation, $P$ contains at least one 1g coin. Two questions:

1. Why is this true?

2. Why do I care?

Let's start with question 2: Why is it helpful to know that $P$ contains a 1g coin?
ThousandthStar 2023-05-23 20:33:48
If $P$ has a 1g coin, we can do the same as with case 1
vrondoS 2023-05-23 20:33:48
then we can remove each coin one at a time
timblack 2023-05-23 20:33:55
Exactly. We can use the same trick as in part 1: We weigh $P$ minus one coin for each coin in $P$. Either one of these weighings gives "at least $X$ grams" which we saw implies exactly $X$ grams, or all of the weighings give "less than $X$ grams," which we saw implies that $P$ minus 1g weighs less than $X$ grams which further implies $P$ weighs exactly $X$ grams.
timblack 2023-05-23 20:34:22
Which brings us back to question 1: If all the $Q_i$ weigh "at least $X$ grams," why does that imply that $P$ contains a 1g coin? We'll proceed by contradiction.
timblack 2023-05-23 20:34:32
Suppose that $P$ consisted entirely of 2g coins. Then, each $Q_i$ has at most one 1g coin: if it had two or more 1g coins, then it would weigh at least 2g less than $P$, so would weigh at most $X + 1 - 2 = X - 1$ grams, but we know that $Q_i$ weighs "at least $X$ grams."
Shivasaurus 2023-05-23 20:34:57
and X-1 is not at least X
timblack 2023-05-23 20:35:02
Exactly.
timblack 2023-05-23 20:35:03
So, each $Q_i$ has at most one 1g coin, so all the $Q_i$ together contain at most $\ell$ coins of 1g, where $\ell < \frac{4046}{k} \leq \frac{4046}{2} = 2023$ coins of 1g. Do you see the contradiction?
timblack 2023-05-23 20:35:49
We're assuming $P$ doesn't contain any 1g coins, so all the $Q_i$ together contain all $2023$ coins of 1g. So the $Q_i$ all together must simultaneously contain less than $2023$ and exactly $2023$ coins of one gram. Contradiction!
timblack 2023-05-23 20:36:03
So, our assumption was false: $P$ must contain at least one 1g coin when every $Q_i$ weighs "at least $X$ grams."
timblack 2023-05-23 20:36:19
That's how we find a set of weigh exactly $X$ in case 2. How many weighings does this take?
timblack 2023-05-23 20:36:27
- It takes $k \leq 2023$ weighings to find $P$.

- It takes $\ell < \frac{4046}{k} \leq \frac{4046}{2} = 2023$ weighings to weigh each $Q_i$.

- It takes $k \leq 2023$ to weigh $P$ one coin for each coin in turn.

So, the total number of weighings is at most $2023 + 2023 + 2023 = 6069 < 10000$.
timblack 2023-05-23 20:36:46
So we're done with case 2, and thus with the problem!
timblack 2023-05-23 20:36:50
:):O
timblack 2023-05-23 20:36:56
The problem only asks you to use less than 10000 weighings. But if you wanted to use even fewer weighings, there are a few things you can do. Any ideas?
NoahLeaf 2023-05-23 20:37:01
yay!
Curious_Droid 2023-05-23 20:37:01
beautiful
Curious_Droid 2023-05-23 20:37:16
binary search?
rithek.shankar 2023-05-23 20:37:16
binary search
GryFry 2023-05-23 20:37:16
binary search
ThousandthStar 2023-05-23 20:37:21
A sort of binary search to find $P$
timblack 2023-05-23 20:37:28
Good idea! For the first step where you find $P$, instead of adding coins one at a time, you can use a "binary search." Line up all $4046$ coins in a row, and let $C_i$ be the set consisting of the first $i$ coins. We want to find the magic number $k$ such that $C_k = P$ weighs "at least $X$ grams," while $C_{k - 1}$ weighs "less than $X$ grams."
timblack 2023-05-23 20:37:43
What we did before was to weigh $C_1$, then $C_2$, then $C_3$, and so on, until we finally got to $k$. We can do this instead: We know our magic $k$ is somwhere between $1$ and $4046$ inclusive. Start right in the middle: weigh $C_{2023}$. If you get "at least $X$", that means lets you narrow down the range of $k$ to somewhere between $1$ and $2023$ inclusive, so you should next try right in the middle of that: weigh $C_{1012}$ next. If you instead are told that $C_{2023}$ weighs "less than X," that tells you that $k$ is somewhere between $2024$ and $4046$, so you should next try the middle of that range: weigh $C_{3035}$ next.
timblack 2023-05-23 20:38:00
With each weighing, you can narrow down the range of possible $k$ by half. If you cut the original range of size $4046$ in half just $12$ times, you can get down to just a single remaining possiblity for $k$. Thus, you can find $k$ in just $12$ weighings instead of $4046$ weighings.
timblack 2023-05-23 20:38:16
Anything else you can do to use fewer weighings?
wilsonstmartin 2023-05-23 20:39:08
Do you need to test 4046 coins? Is 2024 enough?
timblack 2023-05-23 20:39:17
Nice! In case 1, for the second step where you weigh $P$ minus a coin, you don't have to try every coin in $P$. Once you've tried $2024$ coins, you know that at least one of the coins you tried weighs 1g, so you can stop there with only $2024$ weighings for that step instead of up to $4046$.
timblack 2023-05-23 20:39:36
I've thought a bit more about this problem, and I believe that if you take care in a few more places, you can get the number of weighings all the way down to $2026$. But also, $2026$ is the minimum; I believe it is impossible to reliably find a set of weight exactly $X$ in $2025$ weighings. But I haven't written up my solution; feel free to try it for yourself!
timblack 2023-05-23 20:40:01
That concludes problem 3! :):):)
ThousandthStar 2023-05-23 20:40:11
^_^
Solocraftsolo 2023-05-23 20:40:11
Horray!
VanuTalin 2023-05-23 20:40:31
I am ecstatic for Queston 4!!
rithek.shankar 2023-05-23 20:40:31
Problem 4 is weird
VanuTalin 2023-05-23 20:40:31
Number 4 Looks fun!
timblack 2023-05-23 20:40:33
Excellent, on that note, Dan is going to take over to talk about problems 4 through 6!
dgulotta 2023-05-23 20:40:57
Hi everyone!
dgulotta 2023-05-23 20:41:51
4. You are exploring the maze below. The red and blue doors are locked, but you know that there is a Red Key in a treasure chest in one of the rooms, and a Blue Key in a different treasure chest. Once you have the Red Key, you can open all of the red doors. Likewise, once you have the Blue Key, you can open all of the blue doors. https://cdn.aops.com/images/2/0/e/20eaf858e2a35d8d9c50ef091765e952c8634e4b.png
dgulotta 2023-05-23 20:42:15
(a) If I chose two random treasure chests to put the keys in, most likely you will not be able to get to them. Suppose I choose randomly from only the valid assignments: those in which you can get to all of the treasure chests. What is the probability that the Red Key will be in the first treasure chest you open?
dgulotta 2023-05-23 20:42:31
We'd like to enumerate all of the valid assignments - how would you do that?
AoPSyang 2023-05-23 20:43:47
case work
impromptuA 2023-05-23 20:43:47
there are 2 cases, if the red key is in the first room or if it's in the room behind the blue door (1 room away from the first room)
timchen 2023-05-23 20:43:47
Three cases, depending on what keys are in the first room?
GryFry 2023-05-23 20:43:47
case 1: open red door first, case 2: open blue door first
dgulotta 2023-05-23 20:44:37
At least one of the keys has to be in the starting room.
dgulotta 2023-05-23 20:44:57
So they can both be in the starting room, or the red one can be behind the blue door, or the blue one can be behind the red door.
dgulotta 2023-05-23 20:45:52
There are $3 \cdot 2 = 6$ ways of putting both keys in the starting room, $3 \cdot 3 = 9$ ways of putting the red key in the starting room and the blue key in the room behind the red door, and $3 \cdot 2 = 6$ ways of putting the blue key in the starting room and the red key in the room behind the blue door.
dgulotta 2023-05-23 20:46:01
So the fraction of assignments that have the red key in the starting room is $(6+9)/(6+9+6) = 15/21 = 5/7$.
dgulotta 2023-05-23 20:46:31
If the red key is in the starting room, what is the probability that the chest with the red key is the first one you open?
ProCounter44 2023-05-23 20:46:55
1/3
Curious_Droid 2023-05-23 20:46:55
1/3
Rinnypig 2023-05-23 20:46:55
1/3
majix0418 2023-05-23 20:46:55
1/3
UnknownMonkey 2023-05-23 20:46:55
$\frac{1}{3}$
dgulotta 2023-05-23 20:47:08
Right.
rithek.shankar 2023-05-23 20:47:17
So 5/21
dgulotta 2023-05-23 20:47:42
So the overall chance of the red key being in the first chest opened is $5/7 \cdot 1/3 = 5/21$.
Rinnypig 2023-05-23 20:48:08
Yay! Those aren't super great odds.
wilsonstmartin 2023-05-23 20:48:48
Quick question: Is the only way for an assignment to be invalid is if it is behind a door of its own color?
dgulotta 2023-05-23 20:49:12
I think more complicated things could happen, but offhand I'm not 100% sure.
dgulotta 2023-05-23 20:49:27
So that's part (a
winstonDeGreef 2023-05-23 20:49:55
no, there can be a chain where red is behind blue and blue behind red
Malinar 2023-05-23 20:49:55
You could have the blue key behind the red door, and the red key behind the blue door, and none in the starting room - that's an invalid assignment
dgulotta 2023-05-23 20:50:11
(b) Suppose I choose a random key, and put it in a random treasure chest that you can get to. Then I repeat with the other key, but allow it to be placed in any other treasure chest that you can get to after picking up the first key I placed. What is the probability that the Red Key will be in the first treasure chest you open?
dgulotta 2023-05-23 20:50:34
If I place the red key first, I will have to place it in the starting room. If I place the blue key first, what is the probability that the red key will be placed in the starting room?
oyc_1975 2023-05-23 20:51:14
1/2
Curious_Droid 2023-05-23 20:51:14
1/2
impromptuA 2023-05-23 20:51:14
1/2
Malinar 2023-05-23 20:51:14
1/2. There are four chests left, two in the starting room.
dgulotta 2023-05-23 20:51:23
Right.
dgulotta 2023-05-23 20:51:38
So, overall, what is the probability of the red key being in the starting room?
timchen 2023-05-23 20:52:06
$\frac{3}{4}$
winstonDeGreef 2023-05-23 20:52:06
3/4
Malinar 2023-05-23 20:52:06
3/4
dgulotta 2023-05-23 20:52:18
I am equally likely to place the red or blue key first, so the overall probability of the red key being in the starting room is $1/2 \cdot (1 + 1/2) = 3/4$.
dgulotta 2023-05-23 20:52:32
Then the probability of the red key being in the first chest opened is $3/4 \cdot 1/3 = 1/4$.
dgulotta 2023-05-23 20:53:16
So that is part (b).
dgulotta 2023-05-23 20:53:33
(c) The two methods from (a) and (b) give different probabilities, but they're very close together. Can you draw a different map in which the method of (a) gives a probability of picking up the Red Key from the first chest you open of less than 1\%, but the method of (b) gives a probability more than 99\%? Your map may have any number of doors, treasure chests, and colors — but no other types of obstacle, and only one key of each color.
dgulotta 2023-05-23 20:53:43
(To generalize the method from (b), I place the keys in random order, and place the $k$th key in an empty treasure chest you can get to if you have the first $k-1$ keys. If all the treasure chests you can get to are full when I try to place the $k$th key, I take back all the keys placed and start over, once again randomly choosing an order for the keys.)
dgulotta 2023-05-23 20:56:47
First, how might you design the maze so that it is very likely that the method in (b) results in the red key being in the first chest opened?
TheBeast5520 2023-05-23 20:58:22
Initial room has to have at most one treasure box. Otherwise probability is capped at 1/n for both methods (where there are n boxes)
Solocraftsolo 2023-05-23 20:58:32
1 treasure chest in the starting room
wilsonstmartin 2023-05-23 20:59:45
Having a single chest in the first room, and the red key must be in the first room
oyc_1975 2023-05-23 20:59:45
forcing the first key placed to be red, and also having one chest in the starting room
timchen 2023-05-23 20:59:45
other doors have very few chests behind them
JigglypuffDaBest 2023-05-23 20:59:45
make it so that any method that doesn't have red as its first key be a method thats invalid
dgulotta 2023-05-23 20:59:55
There needs to be only one chest in the starting room. Furthermore, if I try to place a non-red key first, it needs to be very likely that I get stuck and have to start over.
dgulotta 2023-05-23 21:00:43
Can you come up with an example maze where the method of (b) gives a very high, but not quite 100%, chance of the red key being in the first chest opened?
dgulotta 2023-05-23 21:00:57
(If the chance was 100%, then the method of (a) would also give 100%.)
dgulotta 2023-05-23 21:02:31
I can have the red door lead to a room with enough chests for all of the keys, and another door that leads to a room with only one chest. For example:
https://cdn.aops.com/images/6/c/8/6c8d08d8103d7c9596e8f67df3171e5af48681f4.png
dgulotta 2023-05-23 21:02:50
If I place the red key first, what is the probability that I succeed in placing all of the keys?
MathFun1000 2023-05-23 21:03:20
$1$
oyc_1975 2023-05-23 21:03:20
1; it will always work
dgulotta 2023-05-23 21:03:24
Right.
dgulotta 2023-05-23 21:03:43
If I place the blue key first, what is the probability?
timchen 2023-05-23 21:04:26
$\frac{1}{100}$
MathFun1000 2023-05-23 21:04:26
$\dfrac{1}{100}$
dgulotta 2023-05-23 21:04:36
$1/100$, since I succeed only if I place the red key second.
dgulotta 2023-05-23 21:05:08
If I place another (not red or blue) key first, what is the probability?
timchen 2023-05-23 21:05:22
0
TheBeast5520 2023-05-23 21:05:22
0
Chameleon07 2023-05-23 21:05:22
0
MathFun1000 2023-05-23 21:05:22
$0$
oyc_1975 2023-05-23 21:05:22
0; you can't place any other keys
dgulotta 2023-05-23 21:05:26
Right.
dgulotta 2023-05-23 21:05:38
So what is the chance that, when I finally succeed in placing the keys, I place the red key in the starting room?
timchen 2023-05-23 21:06:20
$\frac{100}{101}$
impromptuA 2023-05-23 21:06:20
100/101
MathFun1000 2023-05-23 21:06:20
$\tfrac{100}{101}$
dgulotta 2023-05-23 21:06:33
$1/(1 + 1/100 + 99 \cdot 0) = 100/101$, which is greater than 99%
dgulotta 2023-05-23 21:07:23
So we found a way to make it very likely that the method of part (b) will place the red key in the starting room. Now, we'd like to come up with a variation where most of the valid configurations do not place the red key in the starting room. Any ideas on how to do this?
winstonDeGreef 2023-05-23 21:09:11
add another door to the room with one chest and put an enormous amount of chests behind it
dgulotta 2023-05-23 21:09:26
If we have a room that can only be reached using all of the non-red keys, then only the red key can be in that room. If the room is sufficiently huge, then most of the valid assignments will have the red key in that room.
dgulotta 2023-05-23 21:09:46
Here is an example:
https://cdn.aops.com/images/7/e/9/7e9b547121e4cbc9630a83ba32195288ab6aa7c3.png
dgulotta 2023-05-23 21:10:15
First, let's check that the method of part (b) still places the red key in the starting room more than 99% of the time.
dgulotta 2023-05-23 21:12:03
If the first key placed is red, then I'm still guaranteed to be able to place all of the keys, and if it's not red or blue, then I still get stuck right away. If it's blue, what is the probability that I succeed in placing all of the keys?
MathFun1000 2023-05-23 21:12:47
$\tfrac{2}{199}$
dgulotta 2023-05-23 21:12:52
$2/199$, since I succeed only if I place the red or silver key second.
dgulotta 2023-05-23 21:13:20
So the chance that the method of part (b) places the red key in the starting room is $1/(1+2/199+198\cdot 0) = 199/201$.
dgulotta 2023-05-23 21:13:37
Now for the method of part (a). What is an upper bound for the number of valid configurations with the red key in the starting room?
dgulotta 2023-05-23 21:15:16
If the red key is placed in the starting room, then there are $199+1+197=397$ possible spots for the remaining $199$ keys. So the number of ways of placing the remaining keys is at most ${}_{397}P_{199}=\frac{397!}{198!}$.
dgulotta 2023-05-23 21:16:04
How many valid configurations have the red key in the rightmost room?
dgulotta 2023-05-23 21:16:54
There are $\frac{399!}{198!}$ places for the red key, and $197!$ ways to arrange the remaining keys. So there are $\frac{399!}{198!} \cdot 197!$ configurations that have the red key in the rightmost room.
dgulotta 2023-05-23 21:18:22
So there are at least 197! times more configurations with the red key in the rightmost room than in the starting room.
dgulotta 2023-05-23 21:18:43
And $1/(1+197!)$ is a lot less than 1%.
winstonDeGreef 2023-05-23 21:19:19
does |199 chests |red door| start, 1 chest | blue door | 1 chest | grey door| 4 000 000 chests| also work? (again with 200 key colors)
dgulotta 2023-05-23 21:20:50
Hmm, I don't know offhand. That configuration allows the other 197 colors in the large room so it's a different calculation.
JigglypuffDaBest 2023-05-23 21:20:54
can you clarify "1/(1 + 2/199 + 198 * 0)" more? I'm confused how the probabilities from selecting each key ended up in the denominator. . .
dgulotta 2023-05-23 21:22:32
Let's say I attempted to place the keys 200*199 times. I'd expect to attempt to place the red key first 199 times, blue key first 199 times, etc.
dgulotta 2023-05-23 21:22:50
The 199 times with the red key all succeed.
dgulotta 2023-05-23 21:23:01
I only expect to succeed twice with the blue key.
dgulotta 2023-05-23 21:23:06
I never succeed with the other keys.
dgulotta 2023-05-23 21:23:31
So I expect to get 201 successes, 199 of which have the red key in the starting room.
dgulotta 2023-05-23 21:24:18
$1/(1 + 2/199 + 198 * 0)$ is essentially the same calculation, just with the numerator and denominator divided by 200*199.
dgulotta 2023-05-23 21:24:21
Does that make sense?
dgulotta 2023-05-23 21:25:36
Anyway, we have shown that this map is a valid solution to 4c. I'll pause for a bit in case there are any other questions or comments.
dgulotta 2023-05-23 21:28:06
5. Narmada and Travis are playing a game in turns; Narmada goes first. Each player stands on their own number line, which has spaces numbered $1$ through $n$ for some integer $n \ge 3$. Narmada starts at position 1 on her number line, and Travis starts at position n on his. On each player's turn, that player must move to another number on their number line. (The new number need not be adjacent to the old one.) But no repetitions are allowed in the pair of positions: if Narmada moves back to position $1$, then Travis cannot move back to position $n$ on his next move, because the ordered pair $(1,n)$ has already occurred. The first player who cannot move loses. Assume both players play optimally.
dgulotta 2023-05-23 21:28:15
(a) For each $n$, who wins?
dgulotta 2023-05-23 21:28:54
This is a problem where trial and error is useful. I assume many of you have played the game - what did you find?
JigglypuffDaBest 2023-05-23 21:30:04
narmada wins for a
rithek.shankar 2023-05-23 21:30:04
Narmada always won
winstonDeGreef 2023-05-23 21:30:04
narmada can keep moving between 1 and 2 and always have a number to go to
dgulotta 2023-05-23 21:30:27
Narmada can win by moving back and forth between $1$ and $2$.
dgulotta 2023-05-23 21:30:37
Whenever Travis moves to a position $T$, Narmada's next move guarantees that $(1,T)$ and $(2,T)$ are both taken, so Travis never gets an opportunity to move back to $T$. He has to move to a different number each time, and eventually runs out of numbers.
rithek.shankar 2023-05-23 21:30:52
Could we visualize this on the coordinate plane with Narmada as the x-axis and Travis on the y-axis?
Rinnypig 2023-05-23 21:30:52
It is also really helpful to graph this process.
dgulotta 2023-05-23 21:31:16
That's a good point! I'll use some graphs in part c.
dgulotta 2023-05-23 21:31:52
(b) Now suppose they are on the same number line, and cannot simultaneously occupy the same spot. (So if Narmada is at $1$ and Travis is at $3$, Narmada can move to $2, 4, 5, \dotsc, n$ but not $1$ or $3$.) Furthermore, a position is considered the same if the same spots are occupied (so $N=1$, $T=3$ is a repetition of $N=3$, $T=1$). For each $n$, who wins?
dgulotta 2023-05-23 21:32:12
Is the strategy from part (a) helpful here?
Rinnypig 2023-05-23 21:33:00
Yes it is!
rithek.shankar 2023-05-23 21:33:00
Yes, but for Travis
timchen 2023-05-23 21:33:00
kind of, Travis wins because he can cycle back and forth from the bottom to the top
dgulotta 2023-05-23 21:33:16
Now Travis can win by moving back and forth between $n$ and $1$. Narmada can't block him by moving to $1$ or $n$ since $(1,n)$ is already taken. Whenever Narmada moves to a new position $N$, Travis guarantees that $(N,1)$ and $(N,n)$ are both taken, so Narmada can never move back to $N$. She has to move to a new number each time, and eventually runs out of numbers.
dgulotta 2023-05-23 21:34:08
So that solves part (b).
dgulotta 2023-05-23 21:34:21
(c) Same as (b), but the players are considered distinct (so $N=1$, $T=3$ isn't a repetition of $N=3$, $T=1$).
dgulotta 2023-05-23 21:34:38
If you have played a lot of games, you probably noticed something. What jumped out at you?
winstonDeGreef 2023-05-23 21:36:32
for odd n narmada wins
dgulotta 2023-05-23 21:36:41
I'm looking for something slightly more specific.
dgulotta 2023-05-23 21:38:50
Not only does Narmada have a winning strategy, it turns out that *any* strategy is a winning strategy for Narmada! If you run a bunch of simulations with random moves, you will see that she wins every time.
dgulotta 2023-05-23 21:39:01
(for odd n)
dgulotta 2023-05-23 21:39:58
To figure out why Narmada always wins, it's helpful to look at a game more closely. Here is a sample game, shown move-by-move. The $N$ or $T$ in the grid represents the current position ($N$ if it's Narmada's turn, $T$ if it's Travis's). The circles represent positions that have not occurred yet. On Narmada's turn, she moves to a new position in the same row, and on Travis's turn, he moves to a new position in the same column.
dgulotta 2023-05-23 21:40:09
https://www.mathcamp.org/files/yearly/2023/math_jam/narmada-game.png
MathFun1000 2023-05-23 21:40:41
parity matters
Curious_Droid 2023-05-23 21:40:41
narmada has an odd number of squares she can go to at any time, so she can never have 0 squares left to move to
dgulotta 2023-05-23 21:41:02
Yes, Narmada always has an odd number of available moves.
dgulotta 2023-05-23 21:41:16
We'd like to come up with a statement that we can prove by induction, that explains why Narmada always wins. It certainly

seems important that Narmada has an odd number of available moves, but we can't quite use that as an induction hypothesis.

We need to find a stronger statement. Do you notice anything else?
dgulotta 2023-05-23 21:41:28
Here are the counts of the number of unused positions in each row.
dgulotta 2023-05-23 21:41:38
https://cdn.aops.com/images/8/6/9/86970b9166d5c9825a7a31026f4d877ea464e8f0.png
Curious_Droid 2023-05-23 21:43:34
the number of unused positions in every row is even when it is travis' turn?
dgulotta 2023-05-23 21:43:45
Yes. And when it's Narmada's turn?
Curious_Droid 2023-05-23 21:44:20
only the row that she is in has an odd number of unused squares
dgulotta 2023-05-23 21:44:26
Right.
dgulotta 2023-05-23 21:44:38
The counts are even, except that when it's Narmada's turn, the row containing the current position has an odd number of dots.
dgulotta 2023-05-23 21:44:52
This is something that can be proved by induction.
dgulotta 2023-05-23 21:45:40
And in particular, that implies that Narmada always has an odd number of moves, so she can't lose.
dgulotta 2023-05-23 21:47:16
So that solves the odd case.
dgulotta 2023-05-23 21:47:24
Now let's turn our attention to $n$ even. It turns out that if the players play randomly, then either player can win, but Travis seems to win more often.
dgulotta 2023-05-23 21:47:39
Here are two games for $n=4$.

In the first game, Travis wins:
dgulotta 2023-05-23 21:47:51
https://cdn.aops.com/images/2/6/4/264a35fca81e487a99eb9dc64fc2c97d3a8e7500.png
dgulotta 2023-05-23 21:48:09
In the second, Narmada wins:
dgulotta 2023-05-23 21:48:22
https://cdn.aops.com/images/b/7/c/b7ca12b63aec270cd1c188f44f55878023e0411c.png
dgulotta 2023-05-23 21:49:03
What do you notice?
rithek.shankar 2023-05-23 21:51:06
Travis didn't need to lose the second one
rithek.shankar 2023-05-23 21:51:06
He could have moved to 2 instead of 1 on the middle right box.
dgulotta 2023-05-23 21:52:11
We see a pattern similar to the one that we saw in the even case. But there's something weird about position $1$. And moving to position $1$ seems to have cost Travis the game shown above. So what should he do?
JigglypuffDaBest 2023-05-23 21:53:26
he should never move to position 1 because that line is different from others; it had a point already filled out
dgulotta 2023-05-23 21:53:42
If Travis forgets that he's allowed to move to $1$, here's what the first game looks like:
dgulotta 2023-05-23 21:53:55
https://cdn.aops.com/images/3/a/5/3a545ec5c12743b57ffa73ab14841dfd46011c7f.png
dgulotta 2023-05-23 21:54:16
It's the same pattern as before. The count in each column is even, except that when it's Travis's turn, the column containing the current position has an odd number of dots. We can prove by induction that this observation always holds.
dgulotta 2023-05-23 21:54:41
In particular, if Travis forbids himself from moving to $1$, then he always has an odd number of moves available, so he can't lose.
dgulotta 2023-05-23 21:55:40
So we have proved that Narmada wins when $n$ is odd, and Travis wins when $n$ is even.
dgulotta 2023-05-23 21:56:33
Incidentally, a parity argument also works for part (a). If Narmada chooses a subset of $\{1,2,\dotsc,n\}$ of even size and only allows herself to move within that subset, then she always has an odd number of available moves. The strategy that we gave above is just the special case of the subset $\{1,2\}$.
dgulotta 2023-05-23 21:57:33
So that's problem 5. I'll pause for a bit before moving on to problem 6.
dgulotta 2023-05-23 21:59:29
Problem 6: The first quadrant is lava. The rest of the plane (including the $x$ and $y$ axes) is safe. To traverse the lava, you can place a board, which is a line segment of length at most $1$, between two safe endpoints. Once a board is placed, the line segment becomes safe, and points on it may be used as endpoints of a subsequent board. Your goal is to reach the point $(2023, 2023)$.
dgulotta 2023-05-23 21:59:53
(a) Prove that one million boards aren't enough.
dgulotta 2023-05-23 22:00:13
One way to approach this problem is to find a quantity that changes by only a small amount each time you place a board, but needs to change by a large amount in order for $(2023, 2023)$ to be reached. Can you think of such a quantity?
iamhungry 2023-05-23 22:01:00
Area
timchen 2023-05-23 22:01:00
The area under the curve.
impromptuA 2023-05-23 22:01:00
the area of the region surrounded by boards and the x and y axes?
dgulotta 2023-05-23 22:01:25
Yes, it turns out that area is such a quantity.
dgulotta 2023-05-23 22:02:03
So are $\max(xy)$ and the minimum distance squared to (2023,2023). But I'll focus on area, because that is useful for part (c).
rithek.shankar 2023-05-23 22:02:46
what is max(x,y)
dgulotta 2023-05-23 22:03:10
By $max(xy)$, I just mean the largest value of $xy$ among all points on boards.
dgulotta 2023-05-23 22:03:51
Anyway, I should say exactly what I mean by area.



There is an infinite region of lava. When we place a board, it may cut off some lava from the infinite region. We measure the area of the lava cut off from the infinite region.
dgulotta 2023-05-23 22:05:02
In order to bound the area, we first need to make an observation. Does anything stand out to you?
timchen 2023-05-23 22:07:08
We need to reach the square from $(0,0)$ to $(2023,2023)$ to get to that point
winstonDeGreef 2023-05-23 22:07:08
the shape must be concave
Curious_Droid 2023-05-23 22:07:08
the unbounded region is "convex"
dgulotta 2023-05-23 22:09:40
The shape formed by the boards is always concave. We could also prove a weaker statement, that as we go along the boundary, the x-coordinates increase and the y-coordinates decrease.
dgulotta 2023-05-23 22:10:08
How would we prove one of these statements?
iamhungry 2023-05-23 22:10:27
Induction?
dgulotta 2023-05-23 22:11:19
Yes, induction is a good strategy.



If a board cuts off some lava from the infinite region, then it must intersect the previous boundary at two points. As you travel along the previous boundary from one intersection point to the other, the $x$-coordinate is monotone increasing while the $y$-coordinate is monotone decreasing. So the same must be true of the board that you just placed.
dgulotta 2023-05-23 22:12:03
Now how do we bound the area cut off by a single board?
vrondoS 2023-05-23 22:14:24
the area is a triangle
winstonDeGreef 2023-05-23 22:14:24
this area that is added by one board is at most 0.25, which is the maximum area of a right angle triangle with hypotenuse 1 bounding the new area
dgulotta 2023-05-23 22:14:57
Since the boards on the boundary have negative slopes, the region cut off by a single board must be contained in a right triangle with hypotenuse at most $1$. The height of the triangle must be at most $1/2$ (which you can see by inscribing the triangle in a circle). Therefore, the area cut off is at most $(1/2)(1)(1/2) = 1/4$.
dgulotta 2023-05-23 22:15:18
How much area do we need to cut off in order to reach $(2023, 2023)$?
vrondoS 2023-05-23 22:16:13
at least 2023^2
timchen 2023-05-23 22:16:13
$2\cdot2023^2$
winstonDeGreef 2023-05-23 22:16:13
at least 2 * 2023^2
dgulotta 2023-05-23 22:16:28
At the very least, we need to cut off a $2023 \times 2023$ square, which has an area of $2023^2$.
dgulotta 2023-05-23 22:16:59
If you're a bit more careful, you can get $2\cdot 2023^2$, but it's not necessary to solve the problem.
rithek.shankar 2023-05-23 22:17:46
because the slope has to be negative?
dgulotta 2023-05-23 22:17:52
yes
dgulotta 2023-05-23 22:19:15
So we need to fill in an area of at least $2023^2$, but placing a board adds at most $1/4$ to the area. So there's no way to fill the area with 1 million boards.
dgulotta 2023-05-23 22:19:53
Part (b) is somewhat disjoint from part (a), so I'll take questions on part (a) now.
timchen 2023-05-23 22:21:33
How did we rigorously prove that the max area was $\frac{1}{4}?$
dgulotta 2023-05-23 22:22:52
The boards on the boundary have negative slope. When you place a new board, the area cut off is therefore contained in a right triangle with that board as its hypotenuse.
dgulotta 2023-05-23 22:23:09
The hypotenuse has length at most 1.
dgulotta 2023-05-23 22:23:56
The height must be at most 1/2, since the triangle fits in a circle of radius 1/2, with the hypotenuse as a diameter.
Curious_Droid 2023-05-23 22:24:41
comment: I did 6a in the morning half-awake after I woke up with the idea, glad to hear that it was correct lol :P
MasterInTheMaking 2023-05-23 22:24:44
I love problem 6b :D. It was my favorite by far.
irobot 2023-05-23 22:26:03
how can we prove that the slopes are always negative?
dgulotta 2023-05-23 22:26:34
We showed by induction that, as you go along the boundary, the x-coordinates increase while the y-coordinates decrease.
dgulotta 2023-05-23 22:26:46
(b) Give a specific number of boards with which you can reach $(2023, 2023)$.
dgulotta 2023-05-23 22:27:17
We saw quite a few different solutions to this problem. I'll just go over one that is relatively short. If there is interest, I can post additional solutions to parts (a) and (b) in the AoPS forums after the Math Jam is over.
dgulotta 2023-05-23 22:27:33
Let's say that for some $a$, you have made the line $x+y = a$ safe. Any ideas on how to make the line $x+y=a+\epsilon$ safe, for some $\epsilon$?
dgulotta 2023-05-23 22:29:14
https://www.dropbox.com/s/tpb4jnvt3ieojiy/boards.png?dl=1
dgulotta 2023-05-23 22:29:26
You can place a sequence of boards so that each board has one endpoint on the line $x+y=a$, the first board has its other endpoint at $(0,a+1/2)$, and each subsequent board has its other endpoint at the midpoint of the previous board.
dgulotta 2023-05-23 22:29:58
You can do the same thing starting from $(a+1/2,0)$. Eventually, the boards meet in the middle. What is a bound for the number of boards used?
winstonDeGreef 2023-05-23 22:31:09
2a+1
Curious_Droid 2023-05-23 22:31:09
2a?
vrondoS 2023-05-23 22:31:09
<=2a
jbataops 2023-05-23 22:31:57
2sqrt(2) a
timchen 2023-05-23 22:31:57
each board takes up at least $\frac{1}{2}$ space on the boundary, and the total boundary length is $a\sqrt{2},$ so we have $2\sqrt{2}a$
dgulotta 2023-05-23 22:33:10
The bound that I came up with was $(a+1/2)/\sqrt{7/8}$ on each side.
dgulotta 2023-05-23 22:33:52
The first board extends $\sqrt{7/8}$ in the direction parallel to the line, and subsequent boards extend further.
dgulotta 2023-05-23 22:34:51
I won't attempt to simplify it, since I have some numbers pre-computed.
timchen 2023-05-23 22:35:33
Wait why $\sqrt{\frac{7}{8}},$ since if you use the Law of Cosines, then I don't believe that's the answer
dgulotta 2023-05-23 22:38:12
$(0,a+1/2)$ has distance $1/2 *sqrt(2)/2 = sqrt(1/8)$ from the line $x+y=a$, which leaves $sqrt(7/8)$ in the perpendicular direction
shihan 2023-05-23 22:38:46
It doesn't really matter; should we just say it is $\le ka$ where $k<4$?
dgulotta 2023-05-23 22:39:04
Fair enough, the numbers will change slightly, but the argument is the same,
dgulotta 2023-05-23 22:40:10
Anyway, if $a \ge 4046$, then we have reached $(2023,2023)$. So we can assume $a < 4046$.
dgulotta 2023-05-23 22:41:29
Then $(4046+1/2)/\sqrt{7/8} \le 4236$.
dgulotta 2023-05-23 22:41:50
Sorry, 4326.
dgulotta 2023-05-23 22:42:00
So we need at most 4326 boards on each side.
dgulotta 2023-05-23 22:42:16
When the boards meet in the middle, the intersection lies on the line $x+y=a+\epsilon$, for some

$\epsilon$. What is a bound on $\epsilon$?
timchen 2023-05-23 22:43:25
there are approximately a boards on each side, so $\frac{1}{2^a}$?
dgulotta 2023-05-23 22:43:38
The endpoint of the first board is on the line $x+y=a+1/2$, and each board sticks out half as far as the previous one, so we can take $\epsilon = 2^{-4326-1}=2^{-4327}$.
yiningwang 2023-05-23 22:44:07
that is very small
dgulotta 2023-05-23 22:45:15
Now we can place boards along the line $x+y=a+\epsilon$. (However, this isn't strictly necessary - can you see why?)
Curious_Droid 2023-05-23 22:45:52
some boards are actually beyond the line, like the ones at the edges
timchen 2023-05-23 22:45:52
we already made extra progress (a lot, in fact)
dgulotta 2023-05-23 22:46:02
Let's say you have a plan for placing boards along the line $x+y=a$, but instead of the line $x+y=a$ being safe, some chain of segments lying above the line is safe. You can just shorten your boards to end on the safe chain. (Or possibly just not place the board at all, if it lies entirely below the chain.)
dgulotta 2023-05-23 22:46:57
Anyway, each time we execute our plan, we increase $a$ by $2^{-4327}$.
dgulotta 2023-05-23 22:47:37
After $4046 \cdot 2^{-4327}$ implementations, we reach the line $x+y=4046$, which contains $(2023,2023)$.
impromptuA 2023-05-23 22:48:08
$4046*2^{4327}$
dgulotta 2023-05-23 22:48:12
Right, sorry.
dgulotta 2023-05-23 22:48:45
Each implementation uses at most $2 \cdot 4326$ boards.
dgulotta 2023-05-23 22:49:04
So that's a total of $2 \cdot 4326 \cdot 4046 \cdot 2^{4327}$ boards.
dgulotta 2023-05-23 22:49:22
That's quite a lot!
timchen 2023-05-23 22:50:06
Not as much as my count
winstonDeGreef 2023-05-23 22:50:06
I found a way to do it with just 2,644,863,458,071,694
dgulotta 2023-05-23 22:50:24
It's possible to prove a much better bound-the best bound that I know of is around 1.2 trillion.
dgulotta 2023-05-23 22:50:40
I'll post that solution and a bunch of others at the end of the Math Jam.
MasterInTheMaking 2023-05-23 22:51:31
I'd love to see that 1.2 trillion construction!
MasterInTheMaking 2023-05-23 22:51:31
my bound was liek 2^{17128} lol
dgulotta 2023-05-23 22:51:49
I'll pause before moving on to part c.
john0512 2023-05-23 22:52:55
Is there a polynomial upper bound (in terms of the 2023)
dgulotta 2023-05-23 22:53:35
Yes, the number of boards required to reach $(n,n)$ is cubic in $n$.
dgulotta 2023-05-23 22:54:52
One more part to go.
dgulotta 2023-05-23 22:54:54
(c) Prove that one billion boards aren't enough.
dgulotta 2023-05-23 22:55:12
In part (a), we used area. Are there any other quantities that we could consider?
vrondoS 2023-05-23 22:55:36
perimeter?
ninjatutle 2023-05-23 22:55:41
perimeter?
dgulotta 2023-05-23 22:55:55
Let's see what we can learn by considering the perimeter. Again, we need to say what we mean by "perimeter".
dgulotta 2023-05-23 22:56:09
If we place a board from $(\sqrt{2}/2,0)$ to $(0,\sqrt{2}/2)$, then we removed two segments of

length $\sqrt{2}/2$ from the infinite lava region, and added a segment of length $1$. We say that the change in perimeter is $\sqrt{2}/2+\sqrt{2}/2-1 = \sqrt{2}-1$.
dgulotta 2023-05-23 22:56:12
Make sense?
GuylikesMath 2023-05-23 22:56:51
yes
Curious_Droid 2023-05-23 22:56:51
:thumbsup:
ninjatutle 2023-05-23 22:56:51
mhm
dgulotta 2023-05-23 22:57:05
In general, if the boundary is a chain of segments ending at $(0,y)$ and $(x,0)$, then we say that the change in perimeter is $x+y$ minus the length of the chain.
dgulotta 2023-05-23 22:57:19
Assuming the boundary passes through $(2023,2023)$, can you bound the perimeter?
dgulotta 2023-05-23 22:58:31
https://cdn.aops.com/images/3/1/a/31af64d05a7aa211ade3f6475a47a5283ebbed3c.png
dgulotta 2023-05-23 22:58:57
This picture shows that it's at most $2 \cdot 2023$. (Remember, you add the horizontal and vertical segments and subtract the diagonal ones.)
dgulotta 2023-05-23 22:59:47
Previously, we showed that if the boundary passes through $(2023,2023)$, the area must be at least $4 \cdot 2023^2$. So we want to show that if the area is at least $4 \cdot 2023^2$ and the perimeter is at most $2 \cdot 2023$, then more than a billion boards must have been placed.
dgulotta 2023-05-23 23:00:23
Previously, we bounded the area cut off by a single board. Now let's consider the change in perimeter when we place a single board. Let $b$ be the length of the board, let $h$ be the maximum distance from the board to a point in the region cut off.



Let $k$ be the change in area. How large can $k$ be in terms of $b$ and $h$?
Curious_Droid 2023-05-23 23:01:16
change in area or perimeter?
dgulotta 2023-05-23 23:01:30
That was a bit awkwardly worded, but I did mean area - we'll get to perimeter
dgulotta 2023-05-23 23:02:32
The cut off region is contained in a rectangle of length $b$ and height $h$ (again because all slopes are negative). So the area is at most $bh$.
dgulotta 2023-05-23 23:02:49
Let $p$ be the "change in perimeter". The cut off region has one side of length $b$ (the board that was just placed). What is the sum of the lengths of the other sides in terms of $p$ and $b$?
Curious_Droid 2023-05-23 23:04:48
p+b?
dgulotta 2023-05-23 23:04:53
Right.
dgulotta 2023-05-23 23:05:45
We're removing some segments of total length $p+b$ from the boundary, and adding a segment of length $b$, for a difference of $p$.
dgulotta 2023-05-23 23:06:05
Now, can we write down an inequality involving $p$, $b$, and $h$?
dgulotta 2023-05-23 23:06:44
https://cdn.aops.com/images/0/5/a/05adbd26bc3561ba278222fe477f22d504ad250c.png
Curious_Droid 2023-05-23 23:08:00
b+p <= 2h+b? so p <= 2h
Curious_Droid 2023-05-23 23:08:24
$b+p \le \sqrt{b^2 + 4h^2}$
dgulotta 2023-05-23 23:08:41
Almost right, it should be $\ge$.
dgulotta 2023-05-23 23:09:35
$b+p$ is at least the length of the hypotenuse of a right triangle with legs $b$ and $2h$. So $(b+p)^2 \ge b^2 +4h^2$.
dgulotta 2023-05-23 23:09:59
Equivalently, $2bp + p^2 \ge 4h^2$.
dgulotta 2023-05-23 23:10:34
Now, that's not the easiest formula to work with; we'd like something simpler.
dgulotta 2023-05-23 23:11:12
One way to simplify it is to split into two cases, depending on whether $p$ is large or small compared to $b$.
dgulotta 2023-05-23 23:11:45
When $p \le b$, we have $4h^2 \le p^2 + 2pb \le pb + 2pb = 3pb$. So $p \ge 4h^2/3b \ge (4/3)h^2b^2 \ge (4/3)k^2$.
dgulotta 2023-05-23 23:12:24
For $p \ge b$, we have $p \ge b \ge b^4 \ge 16k^2$.
dgulotta 2023-05-23 23:13:19
(Previously, when we showed that $k \le 1/4$, the argument actually proved that $k \le b^2/4$.)
dgulotta 2023-05-23 23:13:42
Combining these two, we find that $p$ is always at least $(4/3)k^2$.
dgulotta 2023-05-23 23:14:03
Does that argument make sense?
JigglypuffDaBest 2023-05-23 23:16:35
this is exactly like one of those crazy solutions they have in AMC 12 #24 or something. . . lol and by saying this, i mean no for my current brain
yiningwang 2023-05-23 23:16:35
no but ok
vrondoS 2023-05-23 23:16:35
yes
dgulotta 2023-05-23 23:17:06
Hmm... I'm happy to take questions at the end, but I guess I'll finish the argument first.
dgulotta 2023-05-23 23:17:24
So, we have found that when we place one board, the change in perimeter $p$ and change in area $k$ satisfy $p \ge (4/3)k^2$.
dgulotta 2023-05-23 23:17:47
Now suppose we place $N$ boards, with total perimeter $P$ and total area $K$,
dgulotta 2023-05-23 23:18:00
By the power mean inequality (or the Cauchy-Schwarz inequality), we get $P/N \ge (4/3)(K/N)^2$. We can rewrite this as $N \ge (4/3) K^2/P$.
dgulotta 2023-05-23 23:18:43
Then $N \ge 4/3 \cdot (2023^2)^2 / (2 \cdot 2023)$, which is over 5 billion.
dgulotta 2023-05-23 23:18:51
That's it!
dgulotta 2023-05-23 23:19:19
It's possible to tweak this argument to get 31 billion.
dgulotta 2023-05-23 23:19:44
That's the end of the quiz. Thank you so much for spending your evening with us!
dgulotta 2023-05-23 23:19:52
If you have any last questions, I can answer them now.
timchen 2023-05-23 23:20:02
Thank you!
Silvermoonleaf 2023-05-23 23:20:02
Thank you!
dgulotta 2023-05-23 23:20:24
We'll be posting tonight's transcript on https://www.mathcamp.org/qualifying_quiz/solutions/ within the next few days, in case you want to revisit some of what we covered tonight.
dgulotta 2023-05-23 23:20:42
I'll also post more solutions to problem 6 at https://artofproblemsolving.com/community/c135t71f135h3069224_math_jam_on_may_23_about_the_2023_qq_problems
dgulotta 2023-05-23 23:21:01
We hope you enjoyed this year's QQ, and stay tuned for the 2024 Quiz (and the summer 2024 application) to be posted in January.
timblack 2023-05-23 23:21:19
Thanks everyone!
devenware 2023-05-23 23:21:31
And a special thank you to our presenters!
snow52 2023-05-23 23:21:42
Thank You!
yiningwang 2023-05-23 23:21:42
bye!!!!
snow52 2023-05-23 23:21:42
Thank you!
JigglypuffDaBest 2023-05-23 23:22:41
Thank you so much!
Joker20 2023-05-23 23:22:41
Thank you!
yiningwang 2023-05-23 23:22:41
thank you!!!

Copyright © 2024 AoPS Incorporated. This page is copyrighted material. You can view and print this page for your own use, but you cannot share the contents of this file with others.